*NURSING > QUESTIONS & ANSWERS > Wayne State University > MCQs Adam Rosh Emergency Medicine: EXAM GRADED A (All)

Wayne State University > MCQs Adam Rosh Emergency Medicine: EXAM GRADED A

Document Content and Description Below

1) A 59-year-old man presents to the emergency department (ED) complaining of new onset chest pain that radiates to his left arm. He has a history of hypertension, hypercholesterolemia, and a 20- pack... -year smoking history. His electrocardiogram (ECG) is remarkable for T-wave inversions in the lateral leads. Which of the following is the most appropriate next step in management? . Give the patient two nitroglycerin tablets sublingually and observe if his chest pain resolves. . Place the patient on a cardiac monitor, administer oxygen, and give aspirin. . Call the cardiac catheterization laboratory for immediate percutaneous intervention (PCI). . Order a chest x-ray; administer aspirin, clopidogrel, and heparin. . Start a β-blocker immediately. A 59-year-old man pr 2) A 36-year-old woman presents to the ED with sudden onset of leftsided chest pain and mild shortness of breath that began the night before. She was able to fall asleep without difficulty but woke up in the morning with persistent pain that is worsened upon taking a deep breath. She walked up the stairs at home and became very short of breath, which made her come to the ED. Two weeks ago, she took a 7-hour flight from Europe and since then has left-sided calf pain and swelling. What is the most common ECG finding for this patient’s presentation? . S1Q3T3 pattern . Atrial fibrillation . Right-axis deviation . Right-atrial enlargement . Tachycardia or nonspecific ST-T–wave changes A 36-year-old woman 3) A 51-year-old man with a long history of hypertension presents to the ED complaining of intermittent chest palpitations lasting for a week. He denies chest pain, shortness of breath, nausea, and vomiting. He recalls feeling similar episodes of palpitations a few months ago but they resolved. His blood pressure (BP) is 130/75 mm Hg, heart rate (HR) is 130 beats per minute, respiratory rate (RR) is 16 breaths per minute, and oxygen saturation is 99% on room air. An ECG is seen below. Which of the following is the most appropriate next step in management? . Sedate patient for immediate synchronized cardioversion with 100 Joules . Prepare patient for the cardiac catheterization laboratory . Administer Coumadin . Administer amiodarone . Administer diltiazem A 51-year-old man wi 4) A 54-year-old woman presents to the ED because of a change in behavior at home. For the past 3 years she has end-stage renal disease requiring dialysis. Her daughter states that the patient has been increasingly tired and occasionally confused for the past 3 days and has not been eating her usual diet. On examination, the patient is alert and oriented to person only. The remainder of her examination is normal. An initial 12-lead ECG is performed as seen below. Which of the following electrolyte abnormalities best explains these findings? . Hypokalemia . Hyperkalemia . Hypocalcemia . Hypercalcemia . Hyponatremia A 54-year-old woman 5) A 29-year-old tall, thin man presents to the ED after feeling short of breath for 2 days. In the ED, he is in no acute distress. His BP is 115/70 mm Hg, HR is 81 beats per minute, RR is 16 breaths per minute, and oxygen saturation is 98% on room air. Cardiac, lung, and abdominal examinations are normal. An ECG reveals sinus rhythm at a rate of 79. A chest radiograph shows a small right-sided (less than 10% of the hemithorax) spontaneous pneumothorax. A repeat chest x-ray 6 hours later reveals a decreased pneumothorax. Which of the following is the most appropriate next step in management? . Discharge the patient with follow-up in 24 hours . Perform needle decompression in the second intercostal space, midclavicular line . Insert a 20F chest tube into right hemithorax . Observe for another 6 hours . Admit for pleurodesis A 29-year-old tall, 6) A 42-year-old man found vomiting in the street is brought to the ED byemergency medical services (EMS). He has a known history of alcohol abuse with multiple presentations for intoxication. Today, the patient complains of acute onset, persistent chest pain associated with dysphagia, and pain upon flexing his neck. His BP is 115/70 mm Hg, HR is 101 beats per minute, RR is 18 breaths per minute, and oxygen saturation is 97% on room air. As you listen to his heart, you hear a crunching sound. His abdomen is soft with mild epigastric tenderness. The ECG is sinus tachycardia without ST-T–wave abnormalities. On chest x-ray, you note lateral displacement of the left mediastinal pleural. What is the most likely diagnosis? . Aspiration pneumonia . Acute pancreatitis . Pericarditis . Esophageal perforation . Aortic dissection A 42-year-old man fo 7) A 65-year-old man with a history of chronic hypertension presents to the ED with sudden-onset tearing chest pain that adiates to his jaw. His BP is 205/110 mm Hg, HR is 90 beats per minute, RR is 20 breaths per minute, and oxygen saturation is 97% on room air. He appears apprehensive. On cardiac examination you hear a diastolic murmur at the right sternal border. A chest x-ray reveals a widened mediastinum. Which of the following is the preferred study of choice to diagnose this patient’s condition? . Electrocardiogram (ECG) . Transthoracic echocardiography (TTE) . Transesophageal echocardiography (TEE) . Computed tomography (CT) scan . Magnetic resonance imaging (MRI) A 65-year-old man wi 8) A 47-year-old man with a history of hypertension presents to the ED complaining of continuous left-sided chest pain that began while snorting cocaine 1 hour ago. The patient states he never experienced chest pain in the past when using cocaine. is BP is 170/90 mm Hg, HR is 101 beats per minute, RR is 18 breaths per minute, and oxygen saturation is 98% on room air. The patient states that the only medication he takes is alprazolam to “calm his nerves.” Which of the following medications is contraindicated in this patient? . Metoprolol . Diltiazem . Aspirin . Lorazepam . Nitroglycerin A 47-year-old man wi 9) A 32-year-old woman presents to the ED with a persistent fever of 101°F over the last 3 days. The patient states that she used to work as a convenience store clerk but was fired 2 weeks ago. Since then, she has been using drugs intravenously daily. Cardiac examination reveals a heart murmur. Her abdomen is soft and nontender with an enlarged spleen. Chest radiograph reveals multiple patchy infiltrates in both lung fields. Laboratory results reveal white blood cells (WBC) 14,000/μL with 91% neutrophils, hematocrit 33%, and platelets 250/μL. An ECG reveals sinus rhythm with first- degree heart block. Which of the following is the most appropriate next step in management? . Obtain four sets of blood cultures, order a transthoracic echocardiogram (TTE), and start antibiotic treatment. . Order a monospot test and recommend that the patient refrain from vigorous activities for 1 month. . Administer a nonsteroidal anti-inflammatory drug (NSAID) and inform the patient she has pericarditis. . Administer isoniazid (INH) and report the patient to the Department of Health. . Order a Lyme antibody and begin antibiotic therapy. A 32-year-old woman 10) A 61-year-old woman was on her way to the grocery store when she started feeling chest pressure in the center of her chest. She became diaphoretic and felt short of breath. On arrival to the ED by EMS, her BP is 130/70 mm Hg, HR is 76 beats per minute, and oxygen saturation is 98% on room air. The nurse gives her an aspirin and an ECG is performed as seen below. Which of the following best describes the location of this patient’s myocardial infarction (MI)? . Anteroseptal . Anterior . Lateral . Inferior . Posterior A 61-year-old woman 11) A 31-year-old man who works for a moving company presents to the ED because he thought he was having a heart attack. He does not smoke, and jogs 3 days a week. His father died of a heart attack in his sixties. He describes a gradual onset of chest pain that is worse with activity and resolves when he is at rest. His HR is 68 beats per minute, BP is 120/70 mm Hg, and RR is 14 breaths per minute. On examination, his lungs are clear and there is no cardiac murmur. You palpate tenderness over the left sternal border at the third and fourth rib. An ECG reveals sinus rhythm at a rate of 65. A chest radiograph shows no infiltrates or pneumothorax. Which of the following is the most appropriate next step in management? . Administer aspirin and send for a troponin. . Administer aspirin, clopidogrel, and heparin and admit for acute coronary syndrome (ACS). . Administer ibuprofen and reassure the patient that he is not having a heart attack. . Inject corticosteroid into the costochondral joint to reduce inflammation. . Observe the patient for 6 hours. A 31-year-old man wh 12) A 21-year-old woman presents to the ED complaining of lightheadedness. Her symptoms appeared 45 minutes ago. She has no other symptoms and is not on any medications. She has a medical history of mitral valve prolapse. Her HR is 170 beats per minute and BP is 105/55 mm Hg. Physical examination is unremarkable. After administering the appropriate medication, her HR slows down and her symptoms resolve. You repeat a 12-lead ECG that shows a rate of 89 beats per minute with a regular rhythm. The PR interval measures 100 msec and there is a slurred upstroke of the QRS complex. Based on this information, which of the following is the most likely diagnosis? . Ventricular tachycardia . Atrial flutter with 3:1 block . Atrial fibrillation . Lown-Ganong-Levine (LGL) syndrome . Wolff-Parkinson-White (WPW) syndrome A 21-year-old woman 13) A 55-year-old man presents to the ED with worsening weakness, muscle cramps, and paresthesias. His past medical history is significant for hypertension and diabetes. He smokes one pack of cigarettes per day. On examination, the patient is alert and oriented and diffusely weak. An ECG is seen below. Which of the following is the most important next step in management? . Administer calcium gluconate. . Administer insulin and dextrose. . Administer aspirin and call the catheterization laboratory. . Order an emergent head CT scan and get a neurology consult. . Collect a sample of his urine to test for ketones. A 55-year-old man pr 14) While eating dinner, a 55-year-old man suddenly feels a piece of steak “get stuck” in his stomach. In the ED, he complains of dysphagia, is drooling ,and occasionally retches. On examination, his BP is 130/80 mm Hg, HR is 75 beats per minute, RR is 15 breaths per minute, and oxygen saturation is 99% on room air. He appears in no respiratory distress. Chest x-ray is negative for air under the diaphragm. Which of the following is the most appropriate next step in management? . Administer 1-mg glucagon intravenously while arranging for endoscopy. . Administer a meat tenderizer such as papain to soften the food bolus. . Administer 10-mL syrup of ipecac to induce vomiting and dislodge the food bolus. . Perform the Heimlich maneuver until the food dislodges. . Call surgery consult to prepare for laparotomy. While eating dinner, 15) A 59-year-old man presents to the ED with left-sided chest pain and shortness of breath that began 2 hours prior to arrival. He states the pain is pressure-like and radiates down his left arm. He is diaphoretic. His BP is 160/80 mm Hg, HR 86 beats per minute, and RR 15 breaths per minute. ECG reveals 2-mm ST-segment elevation in leads I, aVL, V3 to V6. Which of the following is an absolute contraindication to receiving thrombolytic therapy? . Systolic BP greater than 180 mm Hg . Patient on Coumadin and aspirin . Total hip replacement 3 months ago . Peptic ulcer disease . Previous hemorrhagic stroke A 59-year-old man pr 16) A 67-year-old woman is brought to the ED by paramedics complaining of dyspnea, fatigue, and palpitations. Her BP is 80/50 mm Hg, heart is 139 beats per minute, and RR is 20 breaths per minute. Her skin is cool and she is diaphoretic. Her lung examination reveals bilateral crackles and she is beginning to have chest pain. Her ECG shows a narrow complex irregular rhythm with a rate in the 140s. Which of the following is the most appropriate immediate treatment for this patient? . Diltiazem . Metoprolol . Digoxin . Coumadin . Synchronized cardioversion A 67-year-old woman 17) A 61-year-old woman with a history of congestive heart failure (CHF) is at a family picnic when she starts complaining of shortness of breath. Her daughter brings her to the ED where she is found to have an oxygen saturation of 85% on room air with rales halfway up both of her lung fields. Her BP is 185/90 mm Hg and pulse rate is 101 beats per minute. On examination, her jugular venous pressure (JVP) is 6 cm above the sternal angle. There is lower extremity pitting edema. Which of the following is the most appropriate first-line medication to lower cardiac preload? . Metoprolol . Morphine sulfate . Nitroprusside . Nitroglycerin . Oxygen A 61-year-old woman 18) A 27-year-old man who is otherwise healthy presents to the ED with a laceration on his thumb that he sustained while cutting a bagel. You irrigate and repair the wound and are about to discharge the patient when he asks you if he can receive an ECG. It is not busy in the ED so you perform the ECG, as seen below. Which of the following is the most appropriate next step in management? . Admit the patient for placement of a pacemaker. . Admit the patient for a 24-hour observation period. . Administer aspirin and send cardiac biomarkers. . Repeat the ECG because of incorrect lead placement. . Discharge the patient home. A 27-year-old man wh 19) A 56-year-old woman with a history of ovarian cancer presents to the ED with acute onset of right-sided chest pain, shortness of breath, and dyspnea. Her BP is 131/75 mm Hg, HR is 101 beats per minute, respirations are 18 breaths per minute, and oxygen saturation is 97% on room air. You suspect this patient has a pulmonary embolism (PE). Which of the following tests is most likely to be abnormal? . Arterial blood gas . Oxygen saturation . ECG . Chest radiograph . D-dimer A 56-year-old woman 20) A 61-year-old woman with a history of diabetes and hypertension is brought to the ED by her daughter. The patient states that she started feeling short of breath approximately 12 hours ago and then noticed a tingling sensation in the middle of her chest and became diaphoretic. An ECG reveals ST-depression in leads II, III, and aVF. You believe that the patient had a non–ST-elevation MI. Which of the following cardiac markers begins to rise within 3 to 6 hours of chest pain onset, peaks at 12 to 24 hours, and returns to baseline in 7 to 10 days? . Myoglobin . Creatinine kinase (CK) . Creatinine kinase-MB (CK-MB) . Troponin I . Lactic dehydrogenase (LDH) A 61-year-old woman 21) A 27-year-old man complains of chest palpitations and light-headedness for the past hour. He has no past medical history and is not taking any medications. He drinks a beer occasionally on the weekend and does not smoke cigarettes. His HR is 180 beats per minute, BP is 110/65 mm Hg, and oxygen saturation is 99% on room air. An ECG reveals a HR of 180 beats per minute with a QRS complex of 90 msec with a regular rhythm. There are no discernable P waves. Which of the following is the most appropriate medication to treat this dysrhythmia? . Digoxin . Lidocaine . Amiodarone . Adenosine . Bretylium A 27-year-old man co 22) A 59-year-old man presents to the ED with left-sided chest pain and shortness of breath that began 1 hour ago. Initial vital signs are BP 85/45 mm Hg, HR 105 beats per minute, RR 20 breaths per minute, and oxygen saturation 94% on room air. An ECG is seen below. Which of the following is the most appropriate definitive treatment? . Administer metoprolol or diltiazem . Electrical cardio version . Administer calcium gluconate . Thrombolytic therapy . Percutaneous angioplasty A 59-year-old man pr 23) A 55-year-old man presents to the ED at 2:00 AM with left-sided chest pain that radiates down his left arm. He takes a β-blocker for hypertension, a proton-pump inhibitor for gastroesophageal reflux disease, and an antilipid agent for high cholesterol. He also took sildenafil the previous night for erectile dysfunction. His BP is 130/70 mm Hg and HR is 77 beats per minute. Which of the following medication is contraindicated in this patient? . Aspirin . Unfractionated heparin . Nitroglycerin . Metoprolol . Morphine sulfate A 55-year-old man pr 24) A 31-year-old kindergarten teacher presents to the ED complaining of acute onset sub-sternal chest pain that is sharp in nature and radiates to her back. The pain is worse when she is lying down on the stretcher and improves when she sits up. She smokes cigarettes occasionally and was told she has borderline diabetes. She denies any recent surgeries or long travel. Her BP is 145/85 mm Hg, HR is 99 beats per minute, RR is 18 breaths per minute, and temperature is 100.6°F. Examination of her chest reveals clear lungs and a friction rub. Her abdomen is soft and non-tender to palpation. Her legs are not swollen. Chest radiography and echocardiography are unremarkable. Her ECG is shown below. Which of the following is the most appropriate next step in management? . Anticoagulate and CT scan to evaluate for a PE . Prescribe a NSAID and discharge the patient . Aspirin, heparin, clopidogrel, and admit for ACS . Administer thrombolytics if the pain persists . Prescribe antibiotics and discharge the patient A 31-year-old kinder 25) A 71-year-old man is playing cards with some friends when he starts to feel a pain in the left side of his chest. His fingers in the left hand become numb and he feels short of breath. His wife calls the ambulance and he is brought to the hospital. In the ED, an ECG is performed. Which of the following best describes the order of ECG changes seen in an MI? . Hyperacute T wave, ST-segment elevation, Q wave . Q wave, ST-segment elevation, hyperacute T wave . Hyperacute T wave, Q wave, ST-segment elevation . ST-segment elevation, Q wave, hyperacute T wave . ST-segment elevation, hyperacute T wave, Q wave A 71-year-old man is 26) A 63-year-old insurance agent is brought to the ED by paramedics for shortness of breath and a RR of 31 breaths per minute. The patient denies chest pain, fever, vomiting, or diarrhea. His wife says he ran out of his “water pill” 1 week ago. His BP is 185/90 mm Hg, HR is 101 beats per minute, oxygen saturation is 90% on room air, and temperature is 98.9°F. There are crackles midway up both lung fields and 2+ pitting edema midway up his legs. An ECG shows sinus tachycardia. The patient is sitting up and able to speak to you. After placing the patient on a monitor and inserting an IV, which of the following is the most appropriate next step in management? . Obtain blood cultures, complete blood cell (CBC) count, and begin empiric antibiotic therapy. . Order a statim (STAT) portable chest x-ray. . Administer oxygen via nasal cannula and have the patient chew an aspirin. . Administer oxygen via non-rebreather, furosemide, nitroglycerin, and consider noninvasive respiratory therapy. . Rapid sequence endotracheal intubation. A 63-year-old insura 27) A 23-year-old man is brought to the ED after collapsing during a basketball game. His BP is 80/palp. His father died from a “heart-condition” at a young age. He is placed on a monitor and the rhythm is shown below. Which of the following choices best describes this rhythm? . Ventricular fibrillation . Atrial fibrillation . Wolf-Parkinson-White syndrome . Supraventricular tachycardia . Torsades de pointes A 23-year-old man is 28) Which of the following patients has the lowest clinical probability for the diagnosis of pulmonary embolism? . A 21-year-old woman 2 days after a cesarean delivery . A 55-year-old woman on estrogen replacement therapy who underwent a total hip replacement procedure 3 days ago . A 39-year-old man who smokes cigarettes occasionally and underwent an uncomplicated appendectomy 2 months ago . A 62-year-old man with pancreatic cancer . A 45-year-old man with factor V Leiden deficiency Which of the followi 29) A 75-year-old man goes out to shovel snow from his driveway. After 5 minutes of shoveling, he feels short of breath, chest pain, and then passes out. He awakens minutes later to his wife shaking him. In the ED, he denies chest pain or dyspnea. His BP is 160/85 mm Hg, HR is 71 beats per minute, and oxygen saturation is 97% on room air. On examination, you hear a harsh systolic ejection murmur. An ECG reveals a sinus rhythm with left ventricular hypertrophy. Which of the following is the most likely diagnosis? . Asystolic cardiac arrest . Brugada syndrome . Subclavian steal syndrome . PE . Aortic stenosis A 75-year-old man go 30) While playing a match of tennis, a 56-year-old man with a medical history significant only for acid reflux disease starts to feel substernal chest pain that radiates into his left arm and shortness of breath. His pain feels better after drinking antacid, but since it is not completely resolved, his partner calls 911. Upon arrival, EMS administers aspirin and sublingual nitroglycerin. After 20 minutes, the man’s symptoms resolve. He is brought to the ED for further evaluation where his ECG shows sinus rhythm without any ischemic abnormalities. You order a chest radiograph and send his blood work to the laboratory for analysis. Which of the following statements regarding the diagnosis of acute MI is most accurate? . A normal ECG rules out the diagnosis of acute MI. . One set of negative cardiac enzymes is sufficient to exclude the diagnosis of MI in this patient. . Troponin may not reach peak levels for at least 12 hours. . Relief of symptoms by antacids essentially rules out a cardiac cause of his chest pain. . Epigastric discomfort and indigestion is a rare presentation of ACS. While playing a matc 31) A 62-year-old woman presents to the ED with general weakness, shortness of breath, and substernal chest pain that radiates to her left shoulder. Her BP is 155/80 mm Hg, HR is 92 beats per minute, and RR is 16 breaths per minute. You suspect that she is having an acute MI. Which of the following therapeutic agents has been shown to independently reduce mortality in the setting of an acute MI? . Nitroglycerin . Aspirin . Unfractionated heparin . Lidocaine . Diltiazem A 62-year-old woman 32) A 22-year-old college student went to the health clinic complaining of a fever over the last 5 days, fatigue, myalgia’s, and a bout of vomiting and diarrhoea. The clinic doctor diagnosed him with acute gastroenteritis and told him to drink more fluids. Three days later, the student presents to the ED complaining of sub-sternal chest pain that is constant. He also feels short of breath. His temperature is 100.9°F, HR is 119 beats per minute, BP is 120/75 mm Hg, and RR is 18 breaths per minute. An ECG is performed revealing sinus tachycardia. A chest radiograph is unremarkable. Laboratory tests are normal except for slightly elevated WBCs. Which of the following is the most common cause of this patient’s diagnosis? . Streptococcus viridian . Staphylococcus aurous . Coxsackie B virus . Atherosclerotic disease . Cocaine abuse A 22-year-old colleg 33) A 51-year-old woman presents to the ED after 5 consecutive days of crushing substernal chest pressure that woke her up from sleep in the morning. The pain resolves spontaneously after 20 to 30 minutes. She is an avid rock climber and jogs 5 miles daily. She has never smoked cigarettes and has no family history of coronary disease. In the ED, she experiences another episode of chest pain. An ECG reveals ST-segment elevations and cardiac biomarkers are negative. The pain is relieved with sublingual nitroglycerin. She is admitted to the hospital and diagnostic testing reveals minimal coronary atherosclerotic disease. Which of the following is the most appropriate medication to treat this patient’s condition? . Aspirin . Calcium channel blocker (CCB) . β-Blocker . H2-Blocker . Antidepressant A 51-year-old woman 34) A 23-year-old woman who is an elementary school teacher is brought to the ED after syncopizing in her classroom while teaching. Prior to passing out, she describes feeling lightheaded and dizzy and next remembers being in the ambulance. There was no evidence of seizure activity. She has no medical problems and does not take any medications. Her father died of a “heart problem” at 32 years of age. She does not smoke or use drugs. BP is 120/70 mm Hg, pulse rate is 71 beats per minute, RR is 14 breaths per minute, and oxygen saturation is 100% on room air. Her physical examination and laboratory results are all normal. A rhythm strip is seen below. Which of the following is the most likely diagnosis? . Wolff-Parkinson-White syndrome . Long QT syndrome . Lown-Ganong-Levine syndrome . Complete heart block . Atrial flutter A 23-year-old woman 35) While discussing a case presentation with a medical student, a nearby patient who just returned from getting an ankle radiograph done yells out in pain. You walk over to him and ask what is wrong. He states that since returning from the radiology suite, his automatic implantable cardioverter defibrillator (AICD) is discharging. You hook him up to the monitor and note that his rhythm is sinus. You observe a third shock while the patient is in sinus rhythm. Which of the following is the most appropriate next step in management? . Send the patient back to the radiology suite for another radiograph to desensitize his AICD. . Administer pain medication and wait until the device representative arrives at the hospital to power off the AICD. . Admit the patient to the telemetry unit to monitor his rhythm and find the cause of his AICD discharge. . Place a magnet over the AICD generator to inactivate it and thereby prevent further shocks. . Make a small incision over his chest wall and remove the AICD generator and leads. While discussing a c 36) A 55-year-old man presents to the ED with chest pain and shortness of breath. His BP is 170/80 mm Hg, HR is 89 beats per minute, and oxygen saturation is 90% on room air. Physical examination reveals crackles midway up both lung fields and a new holosystolic murmur that is loudest at the apex and radiates to the left axilla. ECG reveals ST elevations in the inferior leads. Chest radiograph shows pulmonary edema with a normalsized cardiac silhouette. Which of the following is the most likely cause of the cardiac murmur? . Critical aortic stenosis . Papillary muscle rupture . Pericardial effusion . CHF . Aortic dissection A 55-year-old man pr 37) An 82-year-old woman is brought to the ED by her daughter for worsening fatigue, dizziness, and lightheadedness. The patient denies chest pain or shortness of breath. She has not started any new medications. Her BP is 140/70 mm Hg, HR is 37 beats per minute, and RR is 15 breaths per minute. An IV is started and blood is drawn. An ECG is seen below. Which of the following is the most appropriate next step in management? (CCU). CCU. . Bed rest for the next 48 hours and follow-up with her primary-care physician. . Administer aspirin, order a set of cardiac enzymes, and admit to the cardiac care unit . Place a magnet on her chest to turn off her pacemaker. . Admit for Holter monitoring and echocardiogram. . Place on a cardiac monitor, place external pacing pads on the patient, and admit to the An 82-year-old woman 38) A 22-year-old man presents to the ED with a history consistent with an acute MI. His ECG reveals ST elevations and his cardiac biomarkers are positive. He has been smoking half a pack of cigarettes per day for the last 3 months. He drinks alcohol when hanging out with his friends. His grandfather died of a heart attack at 80 years of age. The patient does not have hypertension or diabetes mellitus and takes no prescription medications. A recent cholesterol check revealed normal levels of total cholesterol, lowdensity lipoprotein (LDL), and high-density lipoprotein (HDL). Which of the following is the most likely explanation for his presentation? . Cigarette smoking . Family history of heart attack at age 80 years . Incorrectly placed leads on the ECG . Undisclosed cocaine use . Alcohol use A 22-year-old man pr 39) A 29-year-old man is brought to the ED by EMS for a syncopal episode that occurred during a basketball game. A friend states that the patient just dropped to the ground shortly after scoring a basket on a fast-break. On examination, you note a prominent systolic ejection murmur along the left sternal border and at the apex. An ECG reveals left ventricular hypertrophy, left atrial enlargement, and septal Q waves. You suspect the diagnosis and ask the patient to perform the Valsalva maneuver while you auscultate his heart. Which of the following is most likely to occur to the intensity of the murmur with this maneuver? . Decrease . Increase . Remain unchanged . Disappear . The intensity stays the same but the heart skips a beat A 29-year-old man is 40) A 61-year-old woman with metastatic breast cancer presents to the ED with chest pain, cough, and shortness of breath. She states these symptoms began 1 week ago and progressively worsened. She denies fever or chills. On examination, you notice jugular venous distension. Her BP is 105/70 mm Hg and HR is 98 beats per minute. A chest radiograph is shown below. Which of the following ECG finding is associated with this presentation? . Low-voltage complexes . High-voltage complexes . ST-segment depression . ST-segment elevation . T-wave inversion A 61-year-old woman 41) A 68-year-old woman with recently diagnosed uterine cancer is brought to the ED by her daughter. The patient complains of acute onset right-sided chest pain that is sharp in character and worse with inspiration. Her BP is 135/85 mm Hg, HR 107 beats per minute, RR 20 breaths per minute, and oxygen saturation 97% on room air. Physical examination reveals a swollen and tender right calf. ECG is sinus tachycardia. Which of the following is the most appropriate next step in management? . Start heparin therapy prior to diagnostic study . Administer thrombolytics . Order a ventilation-perfusion scan . Order a CT angiogram . Order a D-dimer A 68-year-old woman 42) A 57-year-old man complains of chest palpitations and light-headedness for the past hour. Five years ago he underwent a cardiac catheterization with coronary artery stent placement. He smokes half a pack of cigarettes daily, and drinks a glass of wine at dinner. His HR is 140 beats per minute, BP is 115/70 mm Hg, and oxygen saturation is 99% on room air. An ECG reveals a wide complex tachycardia at a rate of 140 that is regular in rhythm. An ECG from 6 months ago shows a sinus rhythm at a rate of 80. Which of the following is the most appropriate medication to treat this dysrhythmia? . Digoxin . Diltiazem . Amiodarone . Adenosine . Bretylium A 57-year-old man co 43) A 55-year-old man with hypertension and a one-pack-per-day smoking history presents to the ED complaining of three episodes of severe heavy chest pain this morning that radiated to his left shoulder. In the past, he experienced chest discomfort after walking 20 minutes that resolved with rest. The episodes of chest pain this morning occurred while he was reading the newspaper. His BP is 155/80 mm Hg, HR 76 beats per minute, RR 15 breaths per minute. He does not have chest pain in the ED. An ECG reveals sinus rhythm with a rate of 72. A troponin I is negative. Which of the following best describes this patient’s diagnosis? . Variant angina . Stable angina . Unstable angina . Non–ST-elevation MI . ST-elevation MI A 55-year-old man wi 44) A 58-year-old man is brought to the ED for a sync-opal episode at dinner. His wife states that he was well until she found him suddenly slumping in the chair and losing consciousness for a minute. The patient recalls having some chest discomfort and shortness of breath prior to the episode. His rhythm strip, obtained by EMS, is shown below. Which of the following best describes these findings? . Mobitz type I . Mobitz type II . First-degree atrioventricular (AV) block . Atrial flutter with premature ventricular contractions (PVCs) . Sinus bradycardia A 58-year-old man is 45) As you are examining the patient described in the previous question, he starts to complain of chest discomfort and shortness of breath and has another syncopal episode. His ECG is shown below. Which of the following is the most appropriate next step in management? . Call cardiology consult . Cardiovert the patient . Administer metoprolol . Administer amiodarone . Apply transcutaneous pacemaker As you are examining 46) A 55-year-old woman with a past medical history of diabetes walks into the emergency department (ED) stating that her tongue and lips feel like they are swollen. During the history, she tells you that her doctor just started her on a new blood pressure (BP) medication. Her only other medication is a baby aspirin. Her vitals at triage are: BP 130/70 mm Hg, heart rate (HR) 85 beats per minute, respiratory rate (RR) 16 breaths per minute, oxygen saturation 99% on room air, and temperature 98.7°F. On physical examination, you detect mild lip and tongue swelling. Over the next hour, you notice that not only are her tongue and lips getting more swollen, but her face is starting to swell, too. What is the most likely inciting agent? . Metoprolol . Furosemide . Aspirin . Lisinopril . Diltiazem A 55-year-old woman 47) A 45-year-old woman presents to the ED immediately after landing at the airport from a transatlantic flight. She states that a few moments after landing she felt short of breath and felt pain in her chest when she took a deep breath. Her only medications are oral contraceptive pills and levothyroxine. She is a social drinker and smokes cigarettes occasionally. Her BP is 130/75 mm Hg, HR is 98 beats per minute, temperature is 98.9°F, RR is 20 breaths per minute, and oxygen saturation is 97% on room air. You send her for a duplex ultrasound of her legs, which is positive for deep vein thrombosis. What is the most appropriate management for this patient? . Place patient on a monitor, provide supplemental oxygen, and administer unfractionated heparin. . Place patient on a monitor, order a chest computed tomography (CT) scan to confirm a pulmonary embolism (PE), and then administer unfractionated heparin. . Place patient on a monitor and administer aspirin. . Instruct the patient to walk around the ED so that she remains mobile and does not exacerbate thrombus formation. . Place the patient on a monitor, provide supplemental oxygen, and administer warfarin. A 45-year-old woman 48) A 54-year-old undomiciled woman presents to the ED with severe cough, general malaise, and subjective fevers for the last week. She also describes coughing up “chicken livers” during this time and reports that her symptoms are getting progressively worse. Her initial vitals include a HR of 100 beats per minute, a BP of 145/66 mm Hg, temperature of 99.9°F, and an RR of 16 breaths per minute with an oxygen saturation of 95% on room air. She states that she has a history of alcohol abuse, but denies taking any medications or illicit drugs. A chest radiograph shows a lobar pneumonia. Given this patient’s clinical presentation, which of the following is this patient at most risk for contracting? . Streptococcus pneumoniae . Klebsiella pneumoniae . Mycoplasma pneumoniae . Legionella pneumophila . Haemophilus influenzae A 54-year-old undomi 49) A tall, thin 18-year-old man presents to the ED with acute onset of dyspnea while at rest. The patient reports sitting at his desk when he felt a sharp pain on the right side of his chest that worsened with inspiration. His past medical history is significant for peptic ulcer disease. He reports taking a 2-hour plane trip a month ago. His initial vitals include a HR of 100 beats per minute, a BP of 120/60 mm Hg, an RR of 16 breaths per minute, and an oxygen saturation of 97% on room air. On physical examination, you note decreased breath sounds on the right side. Which of the following tests should be performed next? . Electrocardiogram (ECG) . D-dimer . Ventilation perfusion scan (V/Q scan) . Upright abdominal radiograph . Chest radiograph A tall, thin 18-year 50) A 46-year-old alcoholic man presents to the ED with cough, fever, and rigors for 2 days. He describes his sputum as rust-colored. His vital signs are: temperature 101.1°F, HR 94, BP 125/75 mm Hg, RR 20, and pulse oxygen of 97% on room air. Auscultation reveals crackles in the left- lower lobe. Chest radiograph is significant for a left-lower lobar infiltrate. Which of the following organism is the most common cause of community acquired bacterial pneumonia? . Haemophilus influenza . Streptococcus pneumoniae . Klebsiella pneumonia . Group A streptococci . Pseudomonas aeruginosa A 46-year-old alcoho 51) A 30-year-old obese woman with no significant past medical history presents to the ED complaining of shortness of breath and coughing up blood-streaked sputum. The patient states that she travelled to Moscow a month ago. Upon returning to the United States, the patient developed a persistent cough associated with dyspnoea. She was seen by a pulmonologist, who diagnosed her with bronchitis and prescribed an inhaler. However, over the following weeks, the patient’s symptoms worsened, and she developed pleuritic chest pain. In the ED, she lets you know that she smokes half a pack per day. Her vitals include a temperature of 99°F, BP of 105/65 mm Hg, HR of 124 beats per minute, RR of 22 breaths per minute, and an oxygen saturation of 94% on room air. Physical examination is non-contributory, except for roles at the left-mid lung. Her ECG reveals sinus tachycardia with large R waves in V1 to V3 and inverted T waves. Given this patient’s history and presentation, what is the most likely etiology of her symptoms? . Mycoplasma pneumonia (“walking” pneumonia) . Q fever pneumonia . Pneumocystis jiroveci pneumonia (PCP) . PE . Acute respiratory distress syndrome (ARDS) A 30-year-old obese 52) A 24-year-old woman is brought to the ED after being found on a nearby street hunched over and in mild respiratory distress. Upon arrival, she is tachypneic at 24 breaths per minute with an oxygen saturation of 97% on face mask oxygen administration. Upon physical examination, the patient appears to be in mild distress with supraclavicular retractions. Scattered wheezing is heard throughout bilateral lung fields. Which of the following medications should be administered first? . Corticosteroids . Magnesium sulfate . Epinephrine . Anticholinergic nebulizer treatment . β2-Agonist nebulizer treatment A 24-year-old woman 53) An 81-year-old woman presents to the ED with acute onset of shortness of breath just before arrival. She refuses to answer questions for the interview, but repeatedly states that she is feeling short of breath. Her initial vitals include a HR of 89 beats per minute, a BP of 168/76 mm Hg, and an RR of 18 breaths per minute with an oxygen saturation of 89% on room air. A portable chest x-ray appears normal. Her physical examination is unremarkable, except for a systolic ejection murmur. Intravenous (IV) access is successfully obtained. After placing the patient on oxygen and a monitor, which of the following should be performed first? . Evaluation of troponin level . Evaluation of D-dimer level . Rectal temperature . Repeat chest x-ray . ECG An 81-year-old woman 54) A 30-year-old man is brought to the ED by emergency medical service (EMS) in respiratory distress. His initial vitals include a HR of 109 beats per minute, a BP of 180/90 mm Hg, and an RR of 20 breaths per minute with an oxygen saturation of 92% on room air. A chest x-ray shows a bilateral diffuse infiltrative process. A subsequent toxicologic screen is positive. Which of the following agents is most likely responsible for this patient’s presentation? . Cannabis . Opioid . Crack cocaine . Methamphetamine . Ethanol A 30-year-old man is 55) A 26-year-old woman presents to the ED with an acute onset of dyspnea after falling down a few steps. The patient denies any loss of consciousness and reports feeling short of breath. Her initial chest x-ray appears normal; however she continues to be symptomatic with stable vital signs. Which of the following procedures should be performed next? . Repeat upright chest x-ray . Inspiratory and expiratory chest radiographs . Chest CT scan . Chest thoracostomy . Chest thoracotomy A 26-year-old woman 56) As you evaluate a patient with shortness of breath, you appreciate decreased breath sounds at the left-lung base. You suspect the patient has a small pleural effusion. Which of the following views of the chest is this small pleural effusion most likely to be detected? . Supine . Lateral decubitus right-side down . Lateral decubitus left-side down . Lateral . Posterior-anterior (PA) As you evaluate a pa 57) A 32-year-old firefighter presents to the ED in acute respiratory distress. He was taken to the ED shortly after extinguishing a large fire in a warehouse. His initial vitals include a HR of 90 beats per minute, a BP of 120/55 mm Hg, and an RR of 18 breaths per minute with an oxygen saturation of 98% on 2-L nasal cannula. An ECG shows a first-degree heart block. Upon physical examination, there are diffuse rhonchi bilaterally. The patient is covered in soot and the hairs in his nares are singed. Given this clinical presentation, which of the following may be responsible for this patient’s respiratory distress? . Reactive airway disease . Foreign body aspiration . Decompression sickness . Thermal burns . Pneumothorax A 32-year-old firefi 58) A 76-year-old man presents to the ED in acute respiratory distress, gasping for breath while on face mask. Paramedics state that he was found on a bench outside of his apartment in respiratory distress. Initial vitals include a HR of 90 beats per minute, a BP of 170/90 mm Hg, and an RR of 33 breaths per minute with an oxygen saturation of 90%. Upon physical examination, the patient is coughing up pink, frothy sputum, has rales two-thirds of the way up both lung fields, and has pitting edema of his lower extremities. A chest radiograph reveals bilateral perihilar infiltrates, an enlarged cardiac silhouette, and a small right-sided pleural effusion. After obtaining IV access and placing the patient on a monitor, which of the following medical interventions is most appropriate? . Morphine sulfate only . Nitroglycerin only . Nitroglycerin and a loop diuretic . Aspirin . Antibiotics A 76-year-old man pr 59) A 67-year-old man is brought to the ED in respiratory distress. His initial vitals include a HR of 112 beats per minute, a BP of 145/88 mm Hg, and an RR of 18 breaths per minute with an oxygen saturation of 92% on room air. He is also febrile at 102°F. After obtaining IV access, placing the patient on a monitor, and administering oxygen via nasal cannula, a chest radiograph is performed and shows patchy alveolar infiltrates with consolidation in the lower lobes. On review of systems, the patient tells you that he had five to six watery bowel movements a day for the last 2 days with a few bouts of emesis. Which of the following infectious etiologies is most likely responsible for the patient’s presentation? . Streptococcus pneumoniae . Haemophilus influenzae . Mycoplasma pneumoniae . Chlamydophila pneumoniae . Legionella pneumophila A 67-year-old man is 60) A 58-year-old man presents to the ED with progressive dyspnea over the course of 1 week. Upon arrival, he is able to speak in full sentences and states that he stopped taking all of his medications recently. Initial vitals include a HR of 92 beats per minute, a BP of 180/100 mm Hg, and an RR of 16 breaths per minute with an oxygen saturation of 94% on room air. Upon physical examination, the patient has bibasilar crackles, jugular venous distention, and pedal edema. Which of the following medication regimens was the patient most likely on? . Loop diuretic only . Aspirin only . Loop diuretic and β-blocker . Calcium channel blocker . Loop diuretic, β-blocker, and angiotensin-converting enzyme (ACE) inhibitor A 58-year-old man pr 61) A 32-year-old woman presents to the ED with a 1-month history of general malaise, mild cough, and subjective fevers. She states that she is human immunodeficiency virus (HIV) positive and her last CD4 count, 6 months ago, was 220. She is not on antiretroviral therapy or any other medications. Initial vitals include a HR of 88 beats per minute, a BP of 130/60 mm Hg, and an RR of 12 breaths per minute with an oxygen saturation of 91% on room air. Her chest radiograph shows bilateral diffuse interstitial infiltrates. Subsequent laboratory tests are unremarkable except for an elevated lactate dehydrogenase level. Given this patient’s history and physical examination, which of the following is the most likely organism responsible for her clinical presentation? . Coccidioides immitis . Mycobacterium tuberculosis . Pneumocystis jiroveci . Mycoplasma pneumoniae . Haemophilus influenzae A 32-year-old woman 62) A 27-year-old woman presents to the ED complaining of an intensely pruritic rash all over her body, abdominal cramping, and chest tightness. She states that 1 hour ago she was at dinner and accidentally ate some shrimp. She has a known anaphylactic allergy to shrimp. Her BP is 115/75 mm Hg, HR is 95 beats per minute, temperature is 98.9°F, RR is 20 breaths per minute, and oxygen saturation is 97% on room air. She appears anxious, and her skin is flushed with urticarial lesions. Auscultation of her lungs reveals scattered wheezes with decreased air entry. Which of the following is the most appropriate next step in management? . Administer oxygen via non-rebreather, place a large-bore IV, begin IV fluids, and administer methylprednisolone intravenously. . Administer oxygen via non-rebreather, place a large-bore IV, begin IV fluids, and administer methylprednisolone and diphenhydramine intravenously. . Administer oxygen via non-rebreather, place a large-bore IV, begin IV fluids, administer methylprednisolone and diphenhydramine intravenously, and give subcutaneous epinephrine. . Administer oxygen via non-rebreather, place a large-bore IV, begin IV fluids, and start aerosolized albuterol. . Administer oxygen via non-rebreather, place a large-bore IV, begin IV fluids, and start aerosolized epinephrine. A 27-year-old woman 63) An 82-year-old woman becomes acutely short of breath while at rest on the rehabilitation unit. She is brought into the ED with an oxygen saturation of 86% on room air and in acute respiratory distress. Her initial ECG is within normal limits and unchanged from a recent previous examination. Her initial chest x-ray is also negative. Upon chest auscultation, there are equal bilateral breath sounds with some scattered rhonchi. Her nurse tells you that 2 days ago she underwent internal fixation of a right-femur fracture and has been on anticoagulant therapy. Given the history and presentation of this patient, what is the most likely etiology of her symptoms? . Venous thromboembolism . Air embolism . Fat embolism . Pulmonary hemorrhage . Rib fracture An 82-year-old woman 64) A 72-year-old man presents to the ED with worsening dyspnea. His initial vitals include a HR of 93 beats per minute, BP of 110/50 mm Hg, and RR of 20 breaths per minute with an oxygen saturation of 88% on room air. The patient appears thin and anxious. He is using accessory muscles to breathe. Despite distant breath sounds, you hear end-expiratory rhonchi and a prolonged expiratory phase. An ECG shows peaked P waves in leads II, III, and aVF. Given this patient’s history and physical examination, which of the following conditions does this patient most likely have? . Chronic bronchitis . Asthma . Emphysema . Congestive heart failure (CHF) . Pneumothorax A 72-year-old man pr 65) A 71-year-old woman presents to the ED after a reported mechanical fall 2 days ago. Her initial vitals include a HR of 55 beats per minute, a BP of 110/60 mm Hg, an RR of 14 breaths per minute, and an oxygen saturation of 96% on room air. The patient does not appear to be taking deep breaths. Her physical examination is significant for decreased breath sounds bilaterally and tenderness to palpation along the right side of her chest. After initial stabilization, which of the following is the diagnostic test of choice for this patient’s condition? . Chest x-ray . Chest CT scan . ECG . Rib radiographs . Thoracentesis A 71-year-old woman 66) A 29-year-old woman presents to the ED for hyperventilation. Her initial vitals include an RR of 28 breaths per minute with an oxygen saturation of 100% on room air. She is able to speak in full sentences and tells you that she cannot breathe and that her hands and feet are cramping up. She denies any trauma, past medical history, or illicit drug use. Chest auscultation reveals clear breath sounds bilaterally. A subsequent chest radiograph is normal. Upon re-evaluation, the patient reports that she is breathing better. Her vitals include an RR of 12 breaths per minute with an oxygen saturation of 100% on room air. Which of the following conditions is most likely the etiology of this patient’s symptoms? . Pneumothorax . Hemopneumothorax . Pleural effusion . Anxiety attack . Asthma exacerbation A 29-year-old woman 67) A 42-year-old man presents to the ED via ambulance after activating EMS for dyspnea. He is currently on an oxygen face mask and was administered one nebulized treatment of a β2-agonist by the paramedics. His initial vitals include an RR of 16 breaths per minute with an oxygen saturation of 96% on room air. The patient appears to be in mild distress with some intercostal retractions. Upon chest auscultation, there are minimal wheezes localized over bilateral lower lung fields. The patient’s symptoms completely resolve after two more nebulizer treatments. Which of the following medications, in addition to a rescue β2-agonist inhaler, should be prescribed for outpatient use? . Magnesium sulfate . EpiPen . Corticosteroids . Cromolyn sodium . Ipratropium A 42-year-old man pr 68) A 22-year-old woman is brought to the ED by paramedics who state that they found the patient hunched over on a park bench arely breathing. The patient is rousable only to painful stimuli. Her initial vitals include a HR of 78 beats per minute, a BP of 125/58 mm Hg, and a respiratory rate of 6 breaths per minute with an oxygen saturation of 94% on 2-L nasal cannula. Upon physical examination, the patient has clear breath sounds bilaterally and no signs of trauma. Her pupils are 2 mm bilaterally and reactive to light. Which of the following agents may be used to restore this patient’s respirations? . Oxygen . Flumazenil . Anticholinergic inhaler treatment . β2-Agonist nebulized treatment . Naloxone A 22-year-old woman 69) A 43-year-old undomiciled man is brought to the ED after being found intoxicated on the street. He is currently rousable and expresses a request to be left alone. Initial vitals include a HR of 92 beats per minute, a BP of 125/80 mm Hg, and an RR of 14 breaths per minute with an oxygen saturation of 93% on room air. His rectal temperature is 101.2°F. A chest radiograph shows infiltrates involving the right-lower lobe. Given this clinical presentation, what initial antibiotic coverage is most appropriate for this patient? . Gram-negative coverage only . Gram-positive coverage only . Broad-spectrum with anaerobic coverage . PCP coverage . Antifungal therapy A 43-year-old undomi 70) A 32-year-old man is brought into the ED by EMS with fever, shortness of breath, and stridor. The patient was treated yesterday in the ED for a viral syndrome. His BP is 90/50 mm Hg, HR is 110 beats per minute, temperature is 101.2°F, and his RR is 28 breaths per minute. A chest radiograph reveals a widened mediastinum. The patient is endotracheally intubated, given a 2-L bolus of normal saline, and started on antibiotics. His BP improves to 110/70 mm Hg and he is transferred to the intensive care unit (ICU). You see a friend that accompanied the patient to the hospital and ask him some questions. You find out that the patient is a drum maker and works with animal hides. What is the most likely organism that is responsible for the patient’s presentation? . Streptococcus pneumoniae . Corynebacterium diphtheriae . Coxiella burnetii . Haemophilus influenzae . Bacillus anthracis A 32-year-old man is 71) A 62-year-old man presents to the ED with gradual dyspnea over the last few weeks. He reports that he is a daily smoker and has not seen a physician in years. Upon physical examination, there are decreased breath sounds on the right as compared to the left. A chest radiograph indicates blunting of the right costophrenic angle with a fluid line. A thoracentesis is performed. Given this patient’s history, which of the following most likely describes his effusion? . Transudative effusion . Exudative effusion . Transudative and exudative effusion . Lactate dehydrogenase < 200 U . Fluid-to-blood protein ratio < 0.5 A 62-year-old man pr 72) A 40-year-old man with a history of untreated HIV for 8 years comes into the ED complaining of cough, fever, and malaise for 3 ays. He is tachypneic and diaphoretic. Chest radiograph reveals bilateral infiltrates. Arterial blood gas (ABG) analysis is significant for a PaO2 of 62 on room air. His chest radiograph is seen below. Which of the following is the most appropriate initial management? . Corticosteroid treatment prior to antibiotic therapy . Immediate treatment with IV trimethoprim/sulfamethoxazole (TMP/SMX) . Administer antibiotics after a rapid sputum Gram stain is obtained . Nebulizer treatment . Racemic epinephrine A 40-year-old man wi 73) A 49-year-old woman presents to the ED with difficulty breathing after a morning jog. Her initial vitals include a HR of 60 beats per minute, a BP of 120/55 mm Hg, and an RR of 20 breaths per minute with an oxygen saturation of 94% on room air. Upon physical examination, the patient appears to be in mild distress with audible wheezing. She is able to speak in partial sentences and states that she occasionally uses an inhaler. Given this patient’s history and physical examination, which of the following measures should be taken next? . Peak expiratory flow . Chest radiograph . β-Natriuretic peptide level . Rectal temperature . ABG A 49-year-old woman 74) An 81-year-old diabetic woman with a history of atrial fibrillation is transferred to your emergency department (ED) from the local nursing home. The note from the facility states that the patient is complaining of abdominal pain, having already vomited once. Her vital signs in the ED are temperature 100.1°F, blood pressure (BP) 105/75 mm Hg, heart rate (HR) 95 beats per minute, and respiratory rate (RR) 18 breaths per minute. You examine the patient and focus on her abdomen. Considering that the patient has not stopped moaning in pain since arriving to the ED, you are surprised to find that her abdomen is soft on palpation. You decide to order an abdominal radiographic series. Which of the findings on plain abdominal film is strongly suggestive of mesenteric infarction? . Sentinel loop of bowel . No gas in the rectum . Presence of an ileus . Pneumatosis intestinalis . Air-fluid levels An 81-year-old diabe 75) A husband and wife present to the ED with 1 day of subjective fever, vomiting, watery diarrhea, and abdominal cramps. They were t a restaurant a day before for dinner and both ate the seafood special, which consisted of raw shellfish. In the ED, they are both tachycardic with temperatures of 99.8°F and 99.6°F for him and her, respectively. Which of the following is responsible for the majority of acute episodes of diarrhea? . Parasites . Viruses . Enterotoxin-producing bacteria . Anaerobic bacteria . Invasive bacteria A husband and wife p 76) A 79-year-old man was being commemorated at an awards dinner for his 50 years of service at the local bank. While eating a steak dinner, he felt food get stuck in his stomach. He drank a glass of water, but shortly thereafter vomited the water up. He is a bit anxious and decides to come to the ED for further evaluation. His BP is 155/70 mm Hg, HR is 98 beats per minute, RR is 18 breaths per minute, and oxygen saturation is 99% on room air. What is the most common area for an esophageal foreign body to lodge in an adult? . Aortic arch . Cricopharyngeus muscle . Tracheal bifurcation . Lower esophageal sphincter . Pyloric sphincter A 79-year-old man wa 77) As you palpate the right upper quadrant (RUQ) of a 38-year-old woman’s abdomen, you notice that she stops her inspiration for a brief moment. During the history, the patient states that over the last 2 days she gets pain in her RUQ that radiates to her back shortly after eating. Her vitals include a temperature of 100.4°F, HR of 95 beats per minute, BP of 130/75 mm Hg, RR of 16 breaths per minute. What is the initial diagnostic modality of choice for this disorder? . Plain film radiograph . Computed tomography (CT) scan . Magnetic resonance imaging (MRI) . Radioisotope cholescintigraphy (HIDA scan) . Ultrasonography As you palpate the r 78) A 31-year-old man from Florida presents to the ED complaining of severe pain that starts in his left flank and radiates to his testicle. The pain lasts for about 1 hour and then improves. He had similar pain last week that resolved spontaneously. He noted some blood in his urine this morning. His BP is 145/75 mm Hg, HR is 90 beats per minute, temperature is 98.9°F, and his RR is 24 breaths per minute. His abdomen is soft and nontender. As you examine the patient, he vomits and has trouble lying still in his stretcher. Which of the following is the most appropriate next step in management? . Call surgery consult to evaluate the patient for appendicitis. . Order an abdominal CT. . Start intravenous (IV) fluids and administer an IV nonsteroidal anti-inflammatory drug (NSAID) and antiemetic. . Perform an ultrasound to evaluate for an abdominal aortic aneurysm (AAA). . Perform an ultrasound to evaluate for testicular torsion. A 31-year-old man fr 79) A 67-year-old man is brought to the ED by emergency medical service (EMS). His wife states that the patient was doing his usual chores around the house when all of a sudden he started complaining of severe abdominal pain. He has a past medical history of coronary artery disease and hypertension. His BP is 85/70 mm Hg, HR is 105 beats per minute, temperature is 98.9°F, and his RR is 18 breaths per minute. On physical examination, he is diaphoretic and in obvious pain. Upon palpating his abdomen, you feel a large pulsatile mass. An electrocardiogram (ECG) reveals sinus tachycardia. You place the patient on a monitor, administer oxygen, insert two large-bore IVs, and send his blood to the laboratory. His BP does not improve after a 1-L fluid bolus. Which of the following is the most appropriate next step in management? . Order a CT scan to evaluate his aorta. . Call the angiography suite and have them prepare the room for the patient. . Order a portable abdominal radiograph. . Call surgery and have them prepare the operating room (OR) for an exploratory laparotomy. . Call the cardiac catheterization laboratory to prepare for stent insertion. A 67-year-old man is 80) A 57-year-old woman presents to the ED with a basin in her hand and actively vomiting. You insert an IV catheter, start IV fluids, and administer an antiemetic agent. The patient feels much better but also complains of severe crampy abdominal pain that comes in waves. You examine her abdomen and note that it is distended and that there is a small midline scar in the lower abdomen. Upon auscultation, you hear high-pitched noises that sound like “tinkles.” Palpation elicits pain in all four quadrants but no rebound tenderness. She is guaiac negative. Which of the following is the most common cause of this patient’s presentation? . Travel to Mexico . Ethanol abuse . Hysterectomy . Hernia . Constipation A 57-year-old woman 81) An undomiciled 41-year-old man walks into the ED complaining of abdominal pain, nausea, and vomiting. He tells you that he has been drinking beer continuously over the previous 18 hours. On examination, his vitals are BP 150/75 mm Hg, HR 104 beats per minute, RR 16 breaths per minute, oxygen saturation 97% on room air, temperature of 99.1°F rectally, and finger stick glucose 81 mg/dL. The patient is alert and oriented, his pupils anicteric. You notice gynecomastia and spider angiomata. His abdomen is soft but tender in the RUQ. Laboratory tests reveal an AST of 212 U/L, ALT 170 U/L, alkaline phosphatase of 98 U/L, total bilirubin of 1.9 mg/dL, international normalized ratio (INR) of 1.3, WBC 12,000/μL. Urinalysis shows 1+ protein. Chest x-ray is unremarkable. Which of the following is the most appropriate next step in management? . Place a nasogastric tube in the patient’s stomach to remove any remaining ethanol . Order a HIDA scan to evaluate for acute cholecystitis . Administer hepatitis B immune globulin . Send viral hepatitis titers . Supportive care by correcting any fluid and electrolyte imbalances An undomiciled 41-ye 82) A 48-year-old man with a past medical history of hepatitis C and cirrhosis presents to the ED complaining of acute onset abdominal pain and chills. His BP is 118/75 mm Hg, HR is 105 beats per minute, RR is 16 breaths per minute, temperature is 101.2°F rectally, and oxygen saturation is 97% on room air. His abdomen is distended, and diffusely tender. You decide to perform a paracentesis and retrieve 1 L of cloudy fluid. Laboratory analysis of the fluid shows a neutrophil count of 550 cells/mm3. Which of the following is the most appropriate choice of treatment? . Metronidazole . Vancomycin . Sulfamethoxazole/trimethoprim (SMX/TMP) . Neomycin and lactulose . Cefotaxime A 48-year-old man wi 83) A 24-year-old man woke up from sleep 1 hour ago with severe pain in his right testicle. He states that he is sexually active with multiple partners. On examination, the right scrotum is swollen, tender, and firm. You cannot elicit a cremasteric reflex. His BP is 145/75 mm Hg, HR is 103 beats per minute, RR is 14 breaths per minute, temperature is 98.9°F, and oxygen saturation is 99% on room air. Which of the following is the most appropriate next step in management? . Administer one dose of ceftriaxone and doxycycline for 10 days and have him follow-up with a urologist. . Swab his urethra, send a culture for gonorrhea and Chlamydia, and treat if positive. . Send a urinalysis and treat for a urinary tract infection (UTI) if positive. . Treat the patient for epididymitis and have him return if symptoms persist. . Order a statim (STAT) color Doppler ultrasound and urologic consultation. A 24-year-old man wo 84) A 28-year-old man presents to the ED complaining of constant vague, diffuse epigastric pain. He describes having a poor appetite and feeling nauseated ever since eating sushi last night. His BP is 125/75 mm Hg, HR is 96 beats per minute, temperature is 100.5°F, and his RR is 16 breaths per minute. On examination, his abdomen is soft and moderately tender in the right lower quadrant (RLQ). Laboratory results reveal a WBC of 12,000/μL. Urinalysis shows 1+ leukocyte esterase. The patient is convinced that this is food poisoning from the sushi and asks for some antacid. Which of the following is the most appropriate next step in management? . Order a plain radiograph to look for dilated bowel loops. . Administer 40 cc of Maalox and observe for 1 hour. . Send the patient for an abdominal ultrasound. . Order an abdominal CT scan. . Discharge the patient home with ciprofloxacin. A 28-year-old man pr 85) A 41-year-old woman presents to the ED complaining of pain in her RUQ that is steady but gets worse with eating over the past 2 days. The pain also radiates to the right side of her midback. She denies vomiting. Her only medication is an oral contraceptive. Her BP is 140/75 mm Hg, HR is 80 beats per minute, temperature is 98.7°F, and RR is 16 breaths per minute. Laboratory tests are within normal limits. An abdominal ultrasound reveals stones in her gallbladder, but no thickened wall or pericholecystic fluid. What is the most likely diagnosis? . Cholangitis . Urolithiasis . Cholecystitis . Biliary colic . Peptic ulcer disease A 41-year-old woman 86) A 50-year-old man presents to the ED complaining of abrupt onset of epigastric pain that radiates to his back. He describes the pain as constant and associated with nausea and vomiting. The pain improves mildly if he leans forward. He has a 20-pack-year smoking history and has consumed 6 packs of beer daily for more than 5 years. His BP is 150/80 mm Hg, HR is 98 beats per minute, temperature is 100.1°F, and his RR is 18 breaths per minute. He is tender to palpation in the epigastric area. A chest x-ray reveals a small left-sided pleural effusion. His WBC count is 12,000/μL, hematocrit 39%, plasma glucose 225 mg/dL, AST and ALT are within normal limits, alkaline phosphatase 96 U/L, and lipase 520 U/L. His CT scan is seen below. What is the most likely diagnosis? . Abdominal aortic aneurysm . Mesenteric ischemia . Pancreatitis . Bowel perforation . Cholecystitis A 50-year-old man pr 87) A 23-year-old woman presents to the ED in moderate pain in her left lower quadrant (LLQ). She states that the pain began suddenly and is associated with nausea and vomiting. She had a bout of diarrhea yesterday. This is the second time this month that she experienced pain in this location, however, never with this severity. Her BP is 120/75 mm Hg, HR is 101 beats per minute, temperature is 99.5°F, and RR is 18 breaths per minute. She has a tender LLQ on abdominal examination and a tender adnexa on pelvic examination. Which of the following is the most appropriate diagnostic test for the patient? . CT scan . MRI . X-ray . Doppler ultrasound . Laparoscopy A 23-year-old woman 88) A 55-year-old man presents to the ED complaining of mild diffuse abdominal pain. He states that he underwent a routine colonoscopy yesterday and was told “everything is fine.” The pain began upon waking up and is associated with some nausea. He denies fever, vomiting, diarrhea, and rectal bleeding. His BP is 143/71 mm Hg, HR is 87 beats per minute, temperature is 98.9°F, and RR is 16 breaths per minute. His abdomen is tense but only mildly tender. You order baseline laboratory tests. His chest radiograph is seen below. Which of the following is the most likely diagnosis? . Ascending cholangitis . Acute pulmonary edema . Acute liver failure . Pancreatitis . Pneumoperitoneum A 55-year-old man pr 89) A 78-year-old woman is brought to the ED by EMS complaining of vomiting and abdominal pain that began during the night. EMS reports that her BP is 90/50 mm Hg, HR is 110 beats per minute, temperature is 101.2°F, and RR is 18 breaths per minute. After giving her a 500-mL bolus of NS, her BP is 115/70 mm Hg. During the examination, you notice that her face and chest appear jaundiced. Her lungs are clear to auscultation and you do not appreciate a murmur on cardiac examination. She winces when you palpate her RUQ. An ultrasound reveals dilation of the common bile duct and stones in the gallbladder. What is the most likely diagnosis? . Cholecystitis . Acute hepatitis . Cholangitis . Pancreatic cancer . Bowel obstruction A 78-year-old woman 90) A 23-year-old woman presents to the ED complaining of lower abdominal pain and vaginal spotting for 2 days. Her menstrual cycle is irregular. She has a history of ovarian cysts and is sexually active but always uses condoms. Her BP is 115/75 mm Hg, HR is 75 beats per minute, temperature is 98.9°F, and RR is 16 breaths per minute. Which of the following tests should be obtained next? . Chlamydia antigen test . β-Human chorionic gonadotropin (β-hCG) . Transvaginal ultrasound . Abdominal radiograph . Observe her abdominal pain, if it resolves discharge her with a diagnosis of menstruation A 23-year-old woman 91) A 71-year-old obese man is brought to the ED complaining of constant left middle quadrant abdominal pain with radiation into his back. His past medical history is significant for hypertension, peripheral vascular disease, peptic ulcer disease, kidney stones, and gallstones. He smokes a pack of cigarettes and consumes a pint of vodka daily. His BP is 145/80 mm Hg, HR is 90 beats per minute, temperature is 98.9°F, and RR is 16 breaths per minute. Abdominal examination is unremarkable. An ECG is read as sinus rhythm with a HR of 88 beats per minute. An abdominal radiograph reveals normal loops of bowel and curvilinear calcification of the aortic wall. Which of the following is the most likely diagnosis? . Biliary colic . Nephrolithiasis . Pancreatitis . Small bowel obstruction (SBO) . Abdominal aortic aneurysm A 71-year-old obese 92) A 51-year-old man presents to the ED complaining of epigastric pain that radiates to his back. He states that he drinks a 6 pack of beer daily. You suspect he has pancreatitis. His BP is 135/75 mm Hg, HR is 90 beats per minute, temperature is 100.1°F, and his RR is 17 breaths per minute. Laboratory results reveal WBC 13,000/μL, hematocrit 48%, platelets 110/μL, amylase 1150 U/L, lipase 1450 IU, lactate dehydrogenase (LDH) 150 U/L, sodium 135 mEq/L, potassium 3.5 mEq/L, chloride 105 mEq/L, bicarbonate 23 mEq/L, BUN 15 mg/dL, creatinine 1.1 mg/dL, and glucose 125 mg/dL. Which of the following laboratory values are most specific for pancreatitis? . Elevated amylase . Hyperglycemia . Elevated lipase . Elevated LDH . Leukocytosis A 51-year-old man pr 93) A 51-year-old man describes 1 week of gradually worsening scrotal pain and dysuria. He is sexually active with his wife. His temperature is 100.1°F, HR 81 beats per minute, BP 140/75 mm Hg, and oxygen saturation is 99% on room air. On physical examination, his scrotal skin is warm and erythematous. A cremasteric reflex is present. The posterior left testicle is swollen and tender to touch. Color Doppler ultrasonography demonstrates increased testicular blood flow. Urinalysis is positive for leukocyte esterase. What is the most likely diagnosis? . Epididymitis . Testicular torsion . UTI . Testicular tumor . Varicocele A 51-year-old man de 94) A 22-year-old man presents to the ED complaining of dysuria for 3 days. He states that he has never had this feeling before. He is currently sexually active and uses a condom most of the time. He denies hematuria but notes a yellowish discharge from his urethra. His BP is 120/75 mm Hg, HR is 60 beats per minute, and temperature is 98.9°F. You send a clean catch urinalysis to the laboratory that returns positive for leukocyte esterase and 15 white blood cells per high power field (WBCs/hpf). Which of the following is the most appropriate next step in management? . Send a urethral swab for culture and administer 125-mg ceftriaxone intramuscularly and 1- g azithromycin orally. . Send urine for culture and administer SMX/TMP orally. . Discharge the patient with strict instructions to return if his symptoms worsen. . Order a CT scan to evaluate for a kidney stone. . Have him follow-up immediately with a urologist to evaluate for testicular cancer. A 22-year-old man pr 95) A 40-year-old woman presents to the ED complaining of fever and 1 day of increasingly severe pain in her RUQ. She denies nausea or vomiting and has no history of fatty food intolerance. The patient returned from a trip to Mexico 6 months ago. About 2 weeks ago she experienced intermittent diarrhea with blood-streaked mucus. Her BP is 130/80 mm Hg, HR is 107 beats per minute, temperature is 102°F, and RR is 17 breaths per minute. Physical examination reveals decreased breath sounds over the right lung base. Abdominal examination shows tenderness to percussion over the RUQ and normal active bowel sounds. There is no Murphy sign. Her WBC is 20,500/μL. Chest radiograph reveals a small right-pleural effusion. Which of the following is the most likely diagnosis? . Amebic abscess . Cholecystitis . Cryptosporidium . Enterobiasis . Pyogenic abscess A 40-year-old woman 96) A 44-year-old woman is undergoing a diagnostic evaluation for 3 hours of abdominal pain. She had two similar episodes in the past 2 months. She is tolerating oral intake and is afebrile. As part of this evaluation, a diagnostic ultrasound is performed and is shown below. Which of the following is the most likely diagnosis? . Nephrolithiasis . Pancreatic pseudocyst . Ovarian cysts . Cholelithiasis . Liver abscess A 44-year-old woman 97) A 59-year-old woman presents to the ED complaining of worsening lower abdominal pain over the previous 3 days. She describes feeling constipated recently and some burning when she urinates. Her BP is 135/75 mm Hg, HR is 89 beats per minute, temperature is 101.2°F, and her RR is 18 breaths per minute. Her abdomen is mildly distended, tender in the LLQ, and positive for rebound tenderness. CT scan is consistent with diverticulitis with a 7-cm abscess. Which of the following is the most appropriate management for this condition? . Reserve the OR for emergent laparotomy. . Start treatment with ciprofloxacin and metronidazole and plan for CT-guided draining of the abscess. . Give an IV dose of ciprofloxacin and have the patient follow-up with her primary physician. . Start treatment with ciprofloxacin and metronidazole and plan for an emergent barium enema. . Start treatment with ciprofloxacin and metronidazole and prep for an emergent colonoscopy. A 59-year-old woman 98) A 29-year-old man presents to the ED complaining of RLQ pain for 24 hours. He states that the pain first began as a dull feeling around his umbilicus and slowly migrated to his right side. He has no appetite, is nauseated, and vomited twice. His BP is 130/75 mm Hg, HR is 95 beats per minute, temperature is 100.9°F, and his RR is 16 breaths per minute. His WBC is 14,000/μL. As you palpate the LLQ of the patient’s abdomen, he states that his RLQ is painful. What is the name of this sign? . Blumberg sign . Psoas sign . Obturator sign . Raynaud sign . Rovsing sign A 29-year-old man pr 99) A 60-year-old man is brought to the ED complaining of generalized crampy abdominal pain that occurs in waves. He has been vomiting intermittently over the last 6 hours. His BP is 150/75 mm Hg, HR is 90 beats per minute, temperature is 99.8°F, and his RR is 16 breaths per minute. On abdominal examination you notice an old midline scar the length of his abdomen that he states was from surgery after a gunshot wound as a teenager. The abdomen is distended with hyperactive bowel sounds and mild tenderness without rebound. An abdominal plain film confirms your diagnosis. Which of the following is the most appropriate next step in management? . Begin fluid resuscitation, bowel decompression with a nasogastric tube, and request a surgical consult. . Begin fluid resuscitation, administer broad-spectrum antibiotics, and admit the patient to the medical service. . Begin fluid resuscitation, give the patient stool softener, and administer a rectal enema. . Begin fluid resuscitation, administer broad-spectrum antibiotics, and observe the patient for 24 hours. . Order an abdominal ultrasound, administer antiemetics, and provide pain relief. A 60-year-old man is 100) A 73-year-old man who is a 1-pack-per-day smoker and has a medical history of hypertension and peripheral vascular disease presents to the ED complaining of mid-abdominal and right flank pain. He states that he had this same pain 1 week ago and that it got so bad that he passed out. His BP is 125/75 mm Hg, HR is 85 beats per minute, temperature is 98.7°F, and his RR is 17 breaths per minute. Physical examination reveals a bruit over his abdominal aorta and a pulsatile abdominal mass. Which of the following is the most appropriate initial test to evaluate this patient? . Angiography . Ultrasound . MRI . Plain radiograph . D-dimer A 73-year-old man wh 101) A 25-year-old G3P1011 presents to the ED with a 6-hour history of worsening lower abdominal pain, mostly in the RLQ. She also noticed some vaginal spotting this morning. She is nauseated, but did not vomit. Her last menstrual period was 2 months ago, but her cycles are irregular. She is sexually active and has a history of pelvic inflammatory disease. Her BP is 120/75 mm Hg, HR is 95 beats per minute, temperature is 99.2°F, and RR is 16 breaths per minute. Her abdomen is tender in the RLQ. Pelvic examination reveals right adnexal tenderness. Her WBC count is slightly elevated and her β-hCG is positive. After establishing IV access, which of the following is the most appropriate next step in management? . Call the OR to prepare for laparoscopy . Order an emergent CT scan of the abdomen . Perform a transvaginal ultrasound . Order a urinalysis . Swab her cervix and treat for gonorrhea and Chlamydia A 25-year-old G3P101 102) A 59-year-old man presents to the ED complaining of vomiting and sharp abdominal pain in the epigastric area that began abruptly this afternoon. He describes feeling nauseated and has no appetite. Laboratory results reveal WBC 18,000/μL, hematocrit 48%, platelets 110/μL, AST 275 U/L, ALT 125 U/L, alkaline phosphatase 75 U/L, amylase 1150 U/L, lipase 1450 IU, LDH 400 U/L, sodium 135 mEq/L, potassium 3.5 mEq/L, chloride 110 mEq/L, bicarbonate 20 mEq/L, BUN 20 mg/dL, creatinine 1.5 mg/dL, and glucose 250 mg/dL. Which of the following laboratory results correlate with the poorest prognosis? . Amylase 950, lipase 1250, LDH 400 . Lipase 1250, LDH 400, bicarbonate 20 . Lipase 1250, creatinine 1.5, potassium 3.5 . WBC 18,000, LDH 400, glucose 250 . WBC 18,000, amylase 950, lipase 1250 A 59-year-old man pr 103) A 19-year-old woman presents to the ED with 1 hour of acute onset progressively worsening pain in her RLQ. She developed nausea shortly after the pain and vomited twice over the last hour. She had similar but less severe pain 2 weeks ago that resolved spontaneously. Her BP is 123/78 mm Hg, HR is 99 beats per minute, temperature is 99.1°F, and her RR is 16 breaths per minute. On physical examination, the patient appears uncomfortable, not moving on the gurney. Her abdomen is nondistended, diffusely tender, worst in the RLQ. Pelvic examination reveals a normal sized uterus and moderate right-sided adnexal tenderness. Laboratory results reveal WBC 10,000/μL, hematocrit 38%, and a negative urinalysis and β-hCG. Pelvic ultrasound reveals an enlarged right ovary with decreased flow. Which of the following is the most appropriate management for this patient? . Admit to the gynecology service for observation . Administer IV antibiotics and operate once inflammation resolves . Attempt manual detorsion . Order an abdominal CT . Immediate laparoscopic surgery A 19-year-old woman 104) An 18-year-old woman presents to the ED complaining of acute onset of RLQ abdominal pain. She also describes the loss of appetite over the last 12 hours, but denies nausea and vomiting. Her BP is 124/77 mm Hg, HR is 110 beats per minute, temperature is 102.1°F, RR is 16 breaths per minute, and oxygen saturation is 100% on room air. Abdominal examination reveals lower abdominal tenderness bilaterally. On pelvic examination you elicit cervical motion tenderness and note cervical exudates. Her WBC is 20,500/μL and β-hCG is negative. Which of the following is the most appropriate next step in management? . Bring her to the OR for an appendectomy . Begin antibiotic therapy . Perform a culdocentesis . Bring her to the OR for immediate laparoscopy . Order an abdominal plain film An 18-year-old woman 105) A 73-year-old man is seen in the ED for abdominal pain, nausea, and vomiting. His symptoms have progressively worsened over the past 2 to 3 days. The pain is diffuse and comes in waves. He denies fever or chills, but has a history of constipation. He reports no flatus for 24 hours. Physical examination is notable for diffuse tenderness and voluntary guarding. There is no rebound tenderness. An abdominal radiograph is seen below. Which of the following is the most likely diagnosis? . Constipation . SBO . Cholelithiasis . Large bowel obstruction . Inflammatory bowel disease A 73-year-old man is 106) A 25-year-old man presents to the ED complaining of dull peri-umbilical pain that migrated to his RLQ over the last hour. He states that he has no appetite and vomited twice. His BP is 125/75 mm Hg, HR is 87 beats per minute, temperature is 100.6°F, and RR is 16 breaths per minute. Laboratory results reveal WBC 11,000/μL, hematocrit 48%, platelets 170/μL. On physical examination, the patient complains of pain when you flex his knee with internal rotation at his hip. What is the name of this sign? . Obturator . Psoas . Rovsing . McBurney . Murphy A 25-year-old man pr 107) A 27-year-old man is seen in the ED for a leak around a surgical G-tube that was placed 2 weeks ago and has been used for enteral feeding for 1 week. Inspection reveals the tube is pulled out from the stoma, but is still in the cutaneous tissue. The abdomen is soft and non-distended and there are no signs of skin infection. Which of the following is the most appropriate next step in management? . Insert a Foley catheter into the tract and aspirate. If gastric contents are aspirated the tube can be used for feeding. . Insert a Foley catheter into the tract, in-still water-soluble contrast, and obtain an abdominal radiograph prior to using for feeding. . Remove the tube and admit the patient for observation. . Remove the tube and immediately obtain a CT scan of the abdomen. . Return to the OR for closure of gastrostomy and placement of a new tube. A 27-year-old man is 108) A 30-year-old man presents to the ED complaining of sudden onset of abdominal bloating and back pain lasting for 2 days. The pain woke him up from sleep 2 nights ago. It radiates from his back to his abdomen and down toward his scrotum. He is in severe pain and is vomiting. His temperature is 101.2°F and HR is 107 beats per minute. A CT scan reveals a 9-mm obstructing stone of the left ureter with hydronephrosis. Urinalysis is positive for 2+ blood, 2+ leukocytes, 2+ nitrites, 40 to 50 WBCs, and many bacteria. You administer pain medicine, antiemetics, and antibiotics. Which of the following is the most appropriate next step in management? . Admit for IV antibiotics and possible surgical removal of stone. . Observe in ED for another 6 hours to see if stone passes. . Discharge with antibiotics and pain medicine. . Discharge patient with instructions to consume large amounts of water. . Discharge patient with antibiotics, pain medicine, and instructions to drink large amounts of water and cranberry juice. A 30-year-old man pr 109) For which of the following patients is an abdominal CT scan contraindicated? . A 52-year-old man with abdominal pain after blunt trauma, negative focused assessment with sonography for trauma (FAST) examination, BP 125/78 mm Hg, and HR 109 beats per minute . A 22-year-old female with RLQ pain, negative β-hCG, temperature 100.6 F . A 45-year-old man with abdominal pain, temperature 100.5 F, WBC 11,200/μL, BP 110/70 mm Hg, HR 110 beats per minute, and lipase 250 IU . A 70-year-old man with abdominal pain, an 11-cm pulsatile mass in the epigastrium, BP of 70/50 mm Hg, and HR of 110 beats per minute . A 65-year-old woman with right flank pain that radiates to her groin, microhematuria, BP 165/85 mm Hg, and HR 105 beats per minute Abdominal and Pelvic Pain 97 For which of the fol 110) A 71-year-old woman presents to the ED with 12-hours of emesis and abdominal pain. Her temperature is 101.2°F, BP is 100/79 mm Hg, and HR is 104 beats per minute. Physical examination reveals a tender (2 × 2)-cm bulge with erythema below the inguinal ligament and abdominal distension. An occasional high-pitched bowel sound is heard. After placing an IV line and nasogastric tube, which of the following is the most appropriate course of management? . Administer broad-spectrum antibiotics and then obtain a CT scan of abdomen . Administer broad-spectrum antibiotics and attempt reduction . Administer broad-spectrum antibiotics and prepare the patient for the OR . Administer broad-spectrum antibiotics and obtain a plain radiograph . Administer broad-spectrum antibiotics and observe A 71-year-old woman 111) You are working in the ED on a Sunday afternoon when four people present with acute onset vomiting and crampy abdominal pain. They were all at the same picnic and ate most of the same foods. The vomiting began approximately 4 hours into the picnic. They deny having any diarrhea. You believe they may have “food poisoning” so you place IV lines, administer IV fluids, and observe. Over the next few hours, the patients begin to improve, the vomiting stops and their abdominal pain resolves. Which of the following is the most likely cause of their presentation? . Scombroid fish poisoning . Staphylococcal food poisoning . Clostridium perfringens food poisoning . Campylobacter . Salmonellosis You are working in t 112) A 63-year-old man is brought to the ED by EMS complaining of severe abdominal pain that began suddenly 6 hours ago. His BP is 145/75 mm Hg and HR is 105 beats per minute and irregular. On examination, you note mild abdominal distention and diffuse abdominal tenderness without guarding. Stool is heme positive. Laboratory results reveal WBC 12,500/μL, haematocrit 48%, and lactate 4.2 U/L. ECG shows atrial fibrillation at a rate of 110. A CT scan is shown below. Which of the following is the most likely diagnosis? . Abdominal aortic aneurysm . Mesenteric ischemia . Diverticulitis . SBO . Crohn disease A 63-year-old man is 113) A 23-year-old woman presents to the ED with RLQ pain for the last 1 to 2 days. The pain is associated with nausea, vomiting, diarrhea, anorexia, and a fever of 100.9°F. She also reports dysuria. The patient returned 1 month ago from a trip to Mexico. She is sexually active with one partner but does not use contraception. She denies vaginal bleeding or discharge. Her last menstrual period was approximately 1 month ago. She has a history of pyelonephritis. Based on the principles of emergency medicine, what are the three priority considerations in the diagnosis of this patient? . Perihepatitis, gastroenteritis, cystitis . Ectopic pregnancy, appendicitis, pyelonephritis . Pelvic inflammatory disease (PID), gastroenteritis, cystitis . Ectopic pregnancy, PID, menstrual cramps . Gastroenteritis, amebic dysentery, menstrual cramps A 23-year-old woman 114) A 24-year-old woman presents to the ED after being sexually assaulted. She is a college student with no past medical history. Her BP is 130/75 mm Hg, HR is 91 beats per minute, temperature is 98.6°F, and RR is 16 breaths per minute. On physical examination you observe vaginal trauma and scattered bruising and abrasions. Which of the following medications should be offered to the patient in this scenario? . Ceftriaxone, azithromycin, metronidazole, antiretrovirals, emergency contraception . Ceftriaxone, tetanus, metronidazole, antiretrovirals, emergency contraception . Ceftriaxone, azithromycin, tetanus, metronidazole, emergency contraception . Ceftriaxone, azithromycin, tetanus, antiretrovirals, emergency contraception . Ceftriaxone, azithromycin, tetanus, metronidazole, antiretrovirals, emergencycontraception A 24-year-old woman 115) A 71-year-old man presents to the ED with diffuse, crampy abdominal pain that began 1 hour after eating lunch today. The pain is intermittent over the last 8 hours with increasing severity. He also complains of nausea and chills, and vomited once on his way to the ED. He has not had a bowel movement or flatus since the pain began. His past medical history includes prostate cancer, left total hip replacement, appendectomy 25 years ago, right iliac artery aneurysm repair 5 years ago, incisional hernia repair 4 years ago, and irritable bowel syndrome. Which of the following is the most common cause of SBO in adults? . Bezoar . Neoplasm . Incarcerated hernia . Gallstone ileus . Adhesion A 71-year-old man pr 116) An 18-year-old man presents to the ED with nausea and vomiting complaining of testicular pain for the past hour that began while playing volleyball. He recalls having similar pain 1 week ago that resolved spontaneously after 10 minutes. He was recently well and reports no fever, diarrhea, urinary frequency, or dysuria. Physical examination reveals vital signs within normal limits. The patient appears in moderate discomfort, holding his scrotum. His abdomen is soft and non-tender. His right hemiscrotum is swollen, erythematous, and diffusely tender. It is not possible to palpate the testis separate from the epididymis. The right cremasteric reflex is absent. His left testis has a horizontal lie and is non-tender. You suspect testicular torsion. What is the correct way to attempt manual de torsion? . Elevate the painful testis until there is pain relief. . Rotate the testes in a lateral to medial direction as if you were closing a book. . Rotate the testes in a medial to lateral direction as if you were opening a book. . Rotate the testes in an inferior to superior direction. . Rotate the testes in a superior to inferior direction. An 18-year-old man p 117) The following is a radiograph of a 72-year-old man who presented to the ED complaining of gradually worsening back pain that he describes as constant and dull. He denies nausea, vomiting, diarrhea, and hematuria. Which of the following is an important predisposing factor for the development of the condition seen in this individual? . Atherosclerosis . Hyperparathyroidism . Ethanol abuse . Prostate cancer . Hernia The following is a r 118) A 22-year-old woman is brought to the ED by ambulance complaining of sudden onset of severe abdominal pain for 1 hour. The pain is in the RLQ and is not associated with nausea, vomiting, fever, or diarrhea. On the pelvic examination you palpate a tender right adnexal mass. The patient’s last menstrual period was 6 weeks ago. Her BP is 95/65 mm Hg, HR is 124 beats per minute, temperature is 99.8°F, and RR is 20 breaths per minute. Which of the following are the most appropriate next steps in management? . Provide her oxygen via face mask and administer morphine sulfate. . Administer morphine sulfate, order an abdominal CT with contrast, and call an emergent surgery consult. . Send the patient’s urine for analysis and order an abdominal CT. . Bolus 2-L NS, order a type and crossmatch and β-hCG, and call gynecology for possible surgery. . Provide oxygen via face mask, give morphine sulfate, and order a transvaginal ultrasound. A 22-year-old woman 119) A 33-year-old woman presents to the ED complaining of fever, vomiting, and gradually worsening RUQ pain. She states that her pain radiates to her back. Her BP is 130/75 mm Hg, HR is 90 beats per minute, temperature is 100.9°F, and RR is 17 breaths per minute. While examining her abdomen you palpate her RUQ and notice that she momentarily stops her inspiration. What is the name of this classic sign? . Grey-Turner sign . Kernig sign . McMurray sign . Murphy sign . McBurney sign A 33-year-old woman 120) A 21-year-old girl presents to the ED complaining of diarrhea, abdominal cramps, fever, anorexia, and weight loss for 3 days. Her BP is 127/75 mm Hg, HR is 91 beats per minute, and temperature is 100.8°F. Her abdomen is soft and nontender without rebound or guarding. WBC is 9200/μL, β-hCG is negative, urinalysis is unremarkable, and stool is guaiac positive. She tells you that she has had this similar presentation four times over the past 2 months. Which of the following extraintestinal manifestations is associated with Crohn disease but not ulcerative colitis? . Ankylosing spondylitis . Erythema nodosum . Nephrolithiasis . Thromboembolic disease . Uveitis A 21-year-old girl p 121) A 23-year-old woman presents to the ED complaining of pain with urination. She has no other complaints. Her symptoms started 3 weeks ago. During this time, she has been to the clinic twice, with negative urine cultures each time. Her condition has not improved with antibiotic therapy with sulfonamides or quinolones. Physical examination is normal. Which of the following organisms is most likely responsible for the patient’s symptoms? . Staphylococcus aureus . Herpes simplex virus . Trichomonas vaginalis . Escherichia coli . Chlamydia trachomatis A 23-year-old woman 122) A 43-year-old man presents to the ED complaining of progressively worsening abdominal pain over the past 2 days. The pain is constant and radiates to his back. He also describes nausea and vomiting and states he usually drinks 6 pack of beer daily, but has not had a drink for 2 days. His BP is 144/75 mm Hg, HR is 101 beats per minute, temperature is 99.8°F, and RR is 14 breaths per minute. He is lying on his side with his knees flexed. Examination shows voluntary guarding and tenderness to palpation of his epigastrium. Laboratory results reveal WBC 10,500/μL, haematocrit 51%, platelets 225/μL, and lipase 620 IU. An abdominal radiograph reveals a nonspecific bowel gas pattern. There is no free air under the diaphragm. Which of the following is the most appropriate next step in management? . Observe in the ED . Send home with antibiotic therapy . Admit to the hospital for endoscopy . Admit to the hospital for exploratory laparotomy . Admit to the hospital for medical management and supportive care A 43-year-old man pr 123) A 58-year-old woman is brought to the emergency department (ED) by emergency medical service (EMS) after slipping on a patch of ice while walking to work and hitting her head on the cement pavement. Bystanders acknowledged that the patient was unconscious for approximately 1 minute. On arrival, her vital signs are: blood pressure (BP) 155/75 mm Hg, heart rate (HR) 89 beats per minute, respiratory rate (RR) 18 breaths per minute, and pulse oxygenation 98% on room air. She has a 5-cm laceration to the back of her head that is actively bleeding. You ask the patient what happened but she cannot remember. You inform her that she is in the hospital as a result of a fall. Over the next 10 minutes she asks you repeatedly what happened and where she is. You do not find any focal neurologic deficits. As you bring the patient to the CT scanner she vomits once. CT results show a normal brain scan. Which of the following is the most likely diagnosis? . Cerebral concussion . Diffuse axonal injury . Cerebral contusion . Posttraumatic epilepsy . Trauma-induced Alzheimer disease A 58-year-old woman 124) A 41-year-old man, the restrained driver in a high-speed motor vehicle collision, is brought to the ED by EMS. The patient is breathing without difficulty with bilateral and equal breaths sounds. He has strong pulses peripherally indicating a BP of at least 90 mm Hg. The HR is 121 beats per minute. His Glasgow Coma Scale (GCS) is 14. A secondary survey reveals chest wall bruising. You suspect a cardiac injury. Which of the following locations most commonly involve cardiac contusions? . Right atrium . Right ventricle . Left atrium . Left ventricle . Septum A 41-year-old man, t 125) A 25-year-old man is brought into the trauma resuscitation room after his motorcycle is struck by another vehicle. EMS reports that the patient was found 20 ft away from his motorcycle, which was badly damaged. His vital signs include a BP of 90/60 mm Hg, HR of 115 beats per minute, RR of 22 breaths per minute, and pulse oxygenation of 100% on facemask. Which of the following is the smallest amount of blood loss that produces a decrease in the systolic BP in adults? . Loss of 5% of blood volume . Loss of 10% of blood volume . Loss of 15% to 30% of blood volume . Loss of 30% to 40% of blood volume . Loss of greater than 40% of blood volume A 25-year-old man is 126) You are notified by the EMS dispatcher that there is a multiple-car collision on the local highway with many injuries. He states that there are two people dead at the scene, one person is critically injured and hypotensive, and three people have significant injuries, but with stable vital signs. Which of the following is the leading cause of death and disability in trauma victims? . Abdominal injury . Thoracic injury . Back injury . Cervical injury . Head injury You are notified by 127) Paramedics bring a 45-year-old man to the ED after being involved in a high-speed motor vehicle collision. His BP is 85/50 mm Hg and HR is 131 beats per minute after administering 2 L of normal saline. He is awake but slow in responding to questions. A right upper quadrant (RUQ) ultrasound image is seen below. Which of the following is the most appropriate next step in management? . Emergent abdominal computed tomographic (CT) scan . Transfer to the operating room (OR) for laparotomy . Perform a diagnostic peritoneal lavage (DPL) . Observe until one more liter of crystalloid fluid is administered . Serial abdominal examinations Paramedics bring a 4 128) Paramedics bring a 17-year-old high school football player to the ED on a backboard and with a cervical collar. During a football game, the patient “speared” another player with his helmet and subsequently experienced severe neck pain. He denies paresthesias and is able to move all of his extremities. A cervical spine CT scan reveals multiple fractures of the first cervical vertebra. Which of the following best describes this fracture? . Odontoid fracture . Hangman’s fracture . Jefferson fracture . Clay-shoveler’s fracture . Teardrop fracture Paramedics bring a 1 129) A 20-year-old man presents to the ED with multiple stab wounds to his chest. His BP is 85/50 mm Hg and HR is 123 beats per minute. Two large-bore IVs (intravenous) are established and running wide open. On examination, the patient is mumbling incomprehensibly, has good air entry on lung examination, and you notice jugular venous distension. As you are listening to his heart, the nurse calls out that the patient has lost his pulse and that she cannot get a BP reading. Which of the following is the most appropriate next step in management? . Atropine . Epinephrine . Bilateral chest tubes . ED thoracotomy . Pericardiocentesis A 20-year-old man pr 130) A 22-year-old man calls the ED from a local bar stating that he was punched in the face 10 minutes ago and is holding his front incisor tooth in his hand. He wants to know what is the best way to preserve the tooth. Which of the following is the most appropriate advice to give the caller? . Place the tooth in a napkin and bring it to the ED . Place the tooth in a glass of water and bring it to the ED . Place the tooth in a glass of beer and bring it to the ED . Pour some water over the tooth and place it immediately back into the socket . Place the tooth in a glass of milk and bring it to the ED A 22-year-old man ca 131) A 19-year-old man is brought into the trauma room by EMS after a head on cycling accident. The patient was not wearing a helmet. Upon presentation his BP is 125/75 mm Hg, HR is 105 beats per minute, RR is 19 breaths per minute, and oxygen saturation is 100% on mask. His eyes are closed, but open to command. He can move his arms and legs on command. When you ask him questions, he is disoriented but able to converse. What is this patient’s GCS score? . 11 . 12 . 13 . 14 . 15 A 19-year-old man is 132) An 18-year-old man presents to the ED after getting stabbed in his abdomen. His HR is 140 beats per minute and BP is 90/40 mm Hg. He is yelling that he is in pain. Two large-bore IVs are inserted into his antecubital fossa and fluids are running wide open. After 2 L of fluids, his BP does not improve. Which of the following is the most common organ injured in stab wounds? . Liver . Small bowel . Stomach . Colon . Spleen An 18-year-old man p 133) A 61-year-old man presents to the ED with chest wall pain after a motor vehicle collision. He is speaking full sentences, breath sounds are equal bilaterally, and his extremities are well-perfused. His BP is 150/75 mm Hg, HR is 92 beats per minute, and oxygen saturation is 97% on room air. Chest radiography reveals fractures of the seventh and eighth ribs of the right anterolateral chest. He has no other identifiable injuries. Which of the following is the most appropriate treatment for this patient’s rib fractures? . Apply adhesive tape on the chest wall perpendicular to the rib fractures . Insert a chest tube into the right thorax . Bring the patient to the OR for surgical fixation . Analgesia and incentive spirometry . Observation A 61-year-old man pr 134) A 27-year-old man bought to the ED by paramedics after a motor vehicle collision. His RR is 45 breaths per minute, oxygen saturation is 89%, HR is 112 beats per minute, and BP is 115/75 mm Hg. You auscultate his chest and hear decreased breath sounds on the left. Which of the following is the most appropriate next step in management? . Order a STAT chest radiograph . Perform a pericardiocentesis . Perform a DPL . Perform an ED thoracotomy . Perform a tube thoracostomy A 27-year-old man bo 135) A 29-year-old man is brought to the ED by EMS after being stabbed in the left side of his back. His BP is 120/80 mm Hg, HR is 105 beats per minute, RR is 16 breaths per minute, and oxygen saturation is 98% on room air. On the secondary survey, you note motor weakness of his left lower extremity and the loss of pain sensation in the right lower extremity. Which of the following is the most likely diagnosis? . Spinal shock . Central cord syndrome . Anterior cord syndrome . Brown-Séquard syndrome . Cauda equina syndrome A 29-year-old man is 136) A 33-year-old man, who was drinking heavily at a bar, presents to the ED after getting into a fight. A bystander tells paramedics that the patient was punched and kicked multiple times and sustained multiple blows to his head with a stool. In the ED, his BP is 150/75 mm Hg, HR is 90 beats per minute, RR is 13 breaths per minute, and oxygen saturation is 100% on non-rebreather. On examination, he opens his eyes to pain and his pupils are equal and reactive. There is a laceration on the right side of his head. He withdraws his arm to pain but otherwise doesn’t move. You ask him questions, but he just moans. Which of the following is the most appropriate next step in management? . Prepare for intubation . Suture repair of head laceration . Administer mannitol . Bilateral burr holes . Neurosurgical intervention A 33-year-old man, w 137) A 74-year-old man presents to the ED after being involved in a motor vehicle collision. He states he was wearing his seat belt in the driver’s seat when a car hit him from behind. He thinks his chest hit the steering wheel and now complains of pain with breathing. His RR is 20 breaths per minute, oxygen saturation is 98% on room air, BP is 145/75 mm Hg, and HR is 90 beats per minute. On examination, you notice paradoxical respirations. Which of the following best describes a flail chest? . One rib with three fracture sites . Two adjacent ribs each with two fracture sites . Three adjacent ribs each with two fracture sites . One fractured right sided rib and one fractured left-sided rib . Two fractured right sided ribs and two fractured left-sided ribs A 74-year-old man pr 138) A 29-year-old man presents to the ED after being stabbed in his neck. The patient is speaking in full sentences. His breath sounds are equal bilaterally. His BP is 130/75 mm Hg, HR is 95 beats per minute, RR is 16 breaths per minute, and oxygen saturation is 99% on room air. The stab wound is located between the angle of the mandible and the cricoid cartilage and violates the platysma. There is blood oozing from the site although there is no expanding hematoma. Which of the following is the most appropriate next step in management? . Explore the wound and blind clamp any bleeding site. . Probe the wound looking for injured vessels. . Apply direct pressure and bring the patient immediately to the OR to explore the zone 1 injury. . Apply direct pressure and bring the patient immediately to the OR to explore the zone 2 injury. injury. . Apply direct pressure and bring the patient immediately to the OR to explore the zone 3 A 29-year-old man pr 139) A 45-year-old man is brought to the ED after a head-on motor vehicle collision. Paramedics at the scene tell you that the front-end of the car is smashed. The patient’s BP is 130/80 mm Hg, HR is 100 beats per minute, RR is 15 breaths per minute, and oxygen saturation is 98% on room air. Radiographs of the cervical spine reveal bilateral fractures of the C2 vertebra. The patient’s neurologic examination is unremarkable. Which of the following best describes this fracture? . Colles fracture . Boxer’s fracture . Jefferson fracture . Hangman’s fracture . Clay-shoveler’s fracture A 45-year-old man is 140) A 71-year-old man is found lying on the ground one story below the balcony of his apartment. Paramedics bring the patient into the ED. He is cool to touch with a core body temperature of 96°F. His HR is 119 beats per minute and BP is 90/70 mm Hg. His eyes are closed but they open when you call his name. His limbs move to stimuli and he answers your questions but is confused. On examination, you note clear fluid dripping from his left ear canal and an area of ecchymosis around the mastoid bone. Which of the following is the most likely diagnosis? . LeFort fracture . Basilar skull fracture . Otitis interna . Otitis externa . Tripod fracture A 71-year-old man is 141) A 34-year-old construction worker is brought to the ED by EMS after falling 30 ft from a scaffold. His vital signs are HR 124 beats per minute, BP 80/40 mm Hg, and oxygen saturation 93% on 100% oxygen. He has obvious head trauma with a scalp laceration overlying a skull fracture on his occiput. He does not speak when asked his name, his respirations are poor and you hear gurgling with each attempted breath. Auscultation of the chest reveals diminished breath sounds on the right. There is no jugular venous distension (JVD) or anterior chest wall crepitus. His pelvis is unstable with movement laterally to medially and you note blood at the urethral meatus. His right leg is grossly deformed at the knee and there is an obvious fracture of his left arm. Which of the following is the most appropriate next step in management? . Insert a 32F chest tube into the right thoracic cavity. . Perform a DPL to rule out intra-abdominal hemorrhage. . Create two Burr holes into the cranial vault to treat a potential epidural hematoma. . Immediately reduce the extremity injuries and place in a splint until the patient is stabilized. . Plan for endotracheal intubation of the airway with in-line stabilization of the cervical spine. A 34-year-old constr 142) A 20-year-old man was found on the ground next to his car after it hit a tree on the side of the road. Bystanders state that the man got out of his car after the collision but collapsed within a few minutes. Paramedics subsequently found the man unconscious on the side of the road. In the ED, his BP is 175/90 mm Hg, HR is 65 beats per minute, temperature is 99.2°F, RR is 12 breaths per minute, and oxygen saturation is 97% on room air. Physical examination reveals a right-sided fixed and dilated pupil. A head CT is shown below. Which of the following is the most likely diagnosis? . Epidural hematoma . Subdural hematoma . Subarachnoid hemorrhage (SAH) . Intracerebral hematoma . Cerebral contusion A 20-year-old man wa 143) An 81-year-old woman presents to the ED after tripping over the sidewalk curb and landing on her chin causing a hyperextension of her neck. She was placed in a cervical collar by paramedics. On examination, she has no sensorimotor function of her upper extremities. She cannot wiggle her toes, has 1/5 motor function of her quadriceps, and only patchy lower extremity sensation. Rectal examination reveals decreased rectal tone. Which of the following is the most likely diagnosis? . Central cord syndrome . Anterior cord syndrome . Brown-Séquard syndrome . Transverse myelitis . Exacerbation of Parkinson disease An 81-year-old woman 144) A 22-year-old man presents to the ED after being ejected from his vehicle following a high- speed motor vehicle collision. Upon arrival, his BP is 85/55 mm Hg and HR is 141 beats per minute. Two large-bore IVs are placed in the antecubital veins and lactated ringers solution is being administered. After 3 L of crystalloid fluid, the patient’s BP is 83/57 mm Hg. Which of the following statements is most appropriate regarding management of a hypotensive trauma patient who fails to respond to initial volume resuscitation? . It is important to wait for fully cross-matched blood prior to transfusion . Whole blood should be used rather than packed red blood cells (RBCs) . Blood transfusion should begin after 4 L of crystalloid infusion . Type O blood that is Rh-negative should be transfused . Type O blood that is Rh-positive should be transfused A 22-year-old man pr 145) A 24-year-old man is brought into the ED by paramedics after being un over by a car. His systolic BP is 90 mm Hg by palpation, HR is 121 beats per minute, RR is 28 breaths per minute, and oxygen saturation is 100% on non-rebreather. The airway is patent and breath sounds are equal bilaterally. You establish large-bore access and fluids are running wide open. Secondary survey reveals an unstable pelvis upon movement with lateral to medial force. Bedside focused assessment by sonography for trauma (FAST) is negative for intra peritoneal fluid. Which of the following is the most appropriate immediate next step in management? . Bilateral chest tubes . Application of external fixator . Application of pelvic binding apparatus . Venographic embolization . Angiographic embolization A 24-year-old man is 146) A 32-year-old man is brought to the ED by paramedics after a diving accident. The lifeguard on duty accompanies the patient and states that he dove head first into the shallow end of the pool and did not resurface. On examination, the patient is speaking but cannot move his arms or legs and cannot feel pain below his clavicle. He is able to feel light touch and position of his four extremities. A cervical spine radiograph does not reveal a fracture. Which of the following is the most likely diagnosis? . Spinal cord injury without radiographic abnormality (SCIWORA) . Central cord syndrome . Anterior cord syndrome . Cauda equina syndrome . Brown-Séquard syndrome A 32-year-old man is 147) A 22-year-old man is brought to the ED 20 minutes after a head-on motor vehicle collision in which he was the unrestrained driver. On arrival, he is alert and coherent but appears short of breath. His HR is 117 beats per minute, BP is 80/60 mm Hg, and oxygen saturation is 97% on a nonrebreather. Examination reveals bruising over the central portion of his chest. His neck veins are not distended. Breath sounds are present on the left, but absent on the right. Following administration of 2 L of lactated ringer solution, his systolic BP remains at 80 mm Hg. Which of the following is the most appropriate next step in management? . Sedate, paralyze, and intubate . Perform a needle thoracostomy . Perform a DPL . Perform a FAST examination . Perform a pericardiocentesis A 22-year-old man is 148) An 87-year-old man is brought to the ED on a long board and in a cervical collar after falling down a flight of steps. He denies losing consciousness. On arrival, his vital signs include a HR of 99 beats per minute, BP of 160/90 mm Hg, and RR of 16 breaths per minute. He is alert and speaking in full sentences. Breath sounds are equal bilaterally. Despite an obvious right arm fracture, his radial pulses are 2+ and symmetric. When examining his cervical spine, he denies tenderness to palpation and you do not feel any bony deformities. Which of the following is a true statement? . Epidural hematomas are very common in the elderly age population. . Cerebral atrophy in the elderly population provides protection against subdural hematomas. . Increased elasticity of their lungs, allows elderly patients to recover from thoracic trauma more quickly than younger patients. . The most common cervical spine fracture in this age group is a wedge fracture of the sixth cervical vertebra. . Despite lack of cervical spine tenderness, imaging of his cervical spine is warranted. An 87-year-old man i 149) A 41-year-old man who was a restrained front seat passenger in a highspeed motor vehicle collision is brought to the ED by the paramedics. His vital signs are BP 90/50 mm Hg, HR 125 beats per minute, and RR 20 breaths per minute. On examination, he is alert and answers your questions. His breath sounds are equal bilaterally and chest wall is without contusion or bony crepitus. The abdomen is soft and nondistended. His pelvis is unstable. A FAST examination and DPL are negative for intraperitoneal fluid. Initial radiographs reveal a normal chest film and an open book pelvic fracture. Despite infusion of 2 L of lactated ringer solution, his BP is now 80/40 mm Hg. Which of the following is the most appropriate next step in management? . Immediate exploratory laparotomy . Pelvic angiography with embolization of the pelvic vessels . CT scan . Pericardiocentesis . Retrograde urethrogram A 41-year-old man wh 150) A 45-year-old man is brought into the ED after a head-on motor vehicle collision. His BP is 85/45 mm Hg and HR is 130 beats per minute. He is speaking coherently. His breath sounds are equal bilaterally. After 2 L of fluid resuscitation, his BP is 80/40 mm Hg. A FAST examination reveals fluid in Morison pouch. Which of the following organs is most likely to be injured in blunt abdominal trauma? . Liver . Spleen . Kidney . Small bowel . Bladder A 45-year-old man is 151) A 47-year-old man is brought into the ED after falling 20 ft from a ladder. His HR is 110 beats per minute, BP is 110/80 mm Hg, RR is 20 breaths per minute, and oxygen saturation is 100% on face mask. He is able to answer your questions without difficulty. His chest is clear with bilateral breath sounds, abdomen is nontender, pelvis is stable, and the FAST examination is negative. You note a large scrotal hematoma and blood at the urethral meatus. Which of the following is the most appropriate next step in management? . Scrotal ultrasound . Kidney-ureter-bladder (KUB) radiograph . IV pyelogram . Retrograde cystogram . Retrograde urethrogram A 47-year-old man is 152) A 17-year-old man presents to the ED after getting hit in the right eye with a tennis ball during a tennis match. On arrival to the ED, you note periorbital swelling and ecchymosis. The patient’s visual acuity is 20/20. When you are testing his extraocular muscles, you note that his right eye cannot look superiorly but his left eye can. He also describes pain in his right eye when attempting to look upward. Which of the following is the most likely diagnosis? . Zygomatic arch fracture . Orbital floor fracture . Retrobulbar hematoma . Ruptured globe . Mandible fracture with entrapment of the pterygoid A 17-year-old man pr 153) A 24-year-old man is brought to the ED after being shot once in the abdomen. On arrival, his BP is 100/60 mm Hg, HR is 115 beats per minute, and RR is 22 breaths per minute. His airway is patent and you hear breath sounds bilaterally. On abdominal examination, you note a single bullet entry wound approximately 1 cm to the right of the umbilicus. During the log roll, you see a single bullet exit wound approximately 3 cm to the right of the lumbar spine. His GCS score is 15. The patient’s BP is now 85/65 mm Hg and HR is 125 beats per minute after 2 L of fluid. Which of the following is the most appropriate next step in management? . Probe the entry wound to see if it violates the peritoneum . Perform a FAST examination . Perform a DPL . Take the patient directly to the CT scanner . Take the patient directly to the OR A 24-year-old man is 154) A 55-year-old woman presents to the ED stating that her nose has been bleeding profusely for the last 3 hours. After 25 minutes of bilateral pressure on her nasal septum, there is still profuse bleeding. You place anterior nasal packing bilaterally but bleeding still persists. The patient is starting to get anxious. Her BP is 110/70 mm Hg, HR is 80 beats per minute, RR is 18 breaths per minute, and oxygen saturation is 98%. Laboratory results reveal a white blood cell (WBC) count of 9000, hematocrit (HCT) 34%, platelets of 225,000, and international normalized ratio (INR) 1.1. Under direct visualization, you note the bleeding originating from the posterior aspect of her septum. Which of the following is the most appropriate management? . Place posterior nasal packing, start antibiotics, and admit the patient to a monitored hospital bed. . Place the patient supine and wait for spontaneous resolution of the bleeding. . Keep pressure on her nasal septum and administer fresh frozen plasma and platelets. . Place posterior nasal packing, and discharge the patient home with follow-up in 24 hours. . Apply silver nitrate to the nasal mucosa until the bleeding stops. A 55-year-old woman 155) Paramedics bring a 44-year-old man to the ED. He was found in the middle of the street after being struck by a car. His systolic BP is 70 mm Hg; a diastolic BP cannot be obtained. The heart rate is 125 beats per minute, and oxygen saturation is 89% on room air. The patient’s eyes are closed. You ask the patient his name and he doesn’t respond. There is no response when you ask him to move his limbs. You notice that his left foot is severely deformed and there is a large laceration to his right arm. Which of the following is the most appropriate next step in management? . Prepare for emergent orotracheal intubation. . Begin aggressive fluid resuscitation and administer morphine for pain. . Apply a tourniquet just above his left foot and begin fluid resuscitation. . Apply pressure to the laceration, splint the left foot, and order a radiograph. . Administer packed RBCs and bring him to the CT scanner for a pan-scan. Paramedics bring a 4 156) A 17-year-old boy is found unconscious in a swimming pool. He is brought into the ED by paramedics already intubated. In the ED, the patient is unresponsive with spontaneous abdominal breathing at a rate of 16 breaths per minute, BP of 80/50 mm Hg, and HR of 49 beats per minute. In addition to hypoxemia, what condition must be considered earliest in the management of this patient? . Cervical spine injury . Electrolyte imbalance . Metabolic acidosis . Severe atelectasis . Toxic ingestion A 17-year-old boy is 157) A 26-year-old woman, who was a belted front-seat driver in a headon motor vehicle collision, is brought to the ED. She is speaking but complains of progressively worsening shortness of breath and hemoptysis. Her BP is 135/75 mm Hg, HR is 111 beats per minute, RR is 24 breaths per minute, and oxygen saturation is 96% on non-rebreather. On examination, you note ecchymosis over the right side of her chest. Her breath sounds are equal bilaterally. There is no bony crepitus and the trachea is midline. After placing two large-bore IVs and completing the primary and secondary surveys, you view the chest radiograph seen below. Which of the following is the most likely diagnosis? . Diaphragm rupture . Hemothorax . Tension pneumothorax (PTX) . Pulmonary contusion . Acute respiratory distress syndrome (ARDS) A 26-year-old woman, 158) A 22-year-old man is brought to the ED after sustaining a single gunshot wound (GSW) to his right thigh. On arrival, his HR is 105 beats per minute and BP is 115/75 mm Hg. You note a large hematoma of his medial thigh. The patient complains of numbness in his right foot. On extremity examination, the right foot is pale and you cannot palpate a distal pulse but can locate the dorsalis pedis by Doppler. In addition, the patient cannot move the foot. Which of the following is the most appropriate next step in management? . Angiography . Exploration and repair in the OR . Fasciotomy to treat compartment syndrome . Wound exploration . CT scan of the right extremity A 22-year-old man is 159) A 67-year-old woman is brought to the ED after being struck by a cyclist while crossing the street. On arrival to the ED, her eyes remain closed to stimuli, she makes no verbal sounds, and withdraws only to painful stimuli. You assign her a GCS of 6. Her BP is 175/90 mm Hg and HR is 75 beats per minute. As you open her eye lids, you notice that her right pupil is 8 mm and nonreactive and her left is 4 mm and minimally reactive. Which of the following is the most common manifestation of increasing intracranial pressure (ICP) causing brain herniation? . Change in level of consciousness . Ipsilateral pupillary dilation . Contralateral pupillary dilation . Significantly elevated BP . Hemiparesis A 67-year-old woman 160) A 34-year-old man is brought to the ED after being shot in the right side of his chest. The patient is awake and speaking. Breath sounds are diminished on the right. There is no bony crepitus or tracheal deviation. His BP is 95/65 mm Hg, HR is 121 beats per minute, and RR is 23 breaths per minute. Supine chest radiograph reveals a hazy appearance over the entire right lung field. You place a 36F chest tube into the right thoracic cavity and note 1200 cc of blood in the chest tube drainage system. Which of the following is an indication for thoracotomy? . 500 cc of initial chest tube drainage of blood . 1200 cc of initial chest tube drainage of blood . Persistent bleeding from the chest tube at a rate of 50 cc/h . Chest radiograph with greater than 50% lung field white out . Evidence of a PTX A 34-year-old man is 161) A 32-year-old woman is brought to the ED by paramedics after being involved in a motor vehicle collision. The patient was the front-seat passenger of the car and was not wearing a seat belt. In the ED, the patient is speaking and complains of abdominal pain. Her breath sounds are equal bilaterally. You note a distended abdomen. A FAST examination is positive for fluid in the left upper quadrant (LUQ). Her BP is 90/70 mm Hg and HR is 120 beats per minute. You administer 2 L of crystalloid solution. Her repeat BP is 80/60 mm Hg. Which of the following is the most appropriate next step in management? . Administer a vasoconstrictor, such as epinephrine. . Administer another 2 L of crystalloid. . Administer type O, Rh-negative blood. . Bring patient to the CT scanner for an emergent scan. . Perform another FAST examination to see if the fluid is increasing. A 32-year-old woman 162) A 27-year-old pregnant woman, in her third trimester, is brought to the ED after being involved in a low-speed motor vehicle collision. The patient was wearing a seat belt in the back seat of a car that was struck in the front by another car. Her BP is 120/70 mm Hg and HR is 107 beats per minute. Her airway is patent, breath sounds equal bilaterally, and skin is warm with 2+ pulses. FAST examination is negative for free fluid. Evaluation of the fetus reveals appropriate fetal HR and fetal movement. Repeat maternal BP is 120/75 mm Hg. Which of the following is the most appropriate next step in management? . Perform an immediate cesarean section in the OR. . Perform an immediate cesarean section in the ED. . CT scan of the abdomen and pelvis to rule out occult injury. . Discharge the patient if laboratory testing is normal. . Monitor the patient and fetus for a minimum of 4 hours. A 27-year-old pregna 163) A 61-year-old man presents to the ED with low back pain after slipping on an icy sidewalk yesterday. He states that the pain started on the left side of his lower back and now involves the right and radiates down both legs. He also noticed difficulty urinating since last night. On neurologic examination, he cannot plantar flex his feet. Rectal examination reveals diminished rectal tone. He has a medical history of chronic hypertension and underwent a “vessel surgery” many years earlier. Which of the following is the best diagnosis? . Abdominal aortic aneurysm . Disk herniation . Spinal stenosis . Cauda equina syndrome . Osteomyelitis A 61-year-old man pr 164) Paramedics bring a 55-year-old woman to the ED after she was struck by a motor vehicle traveling at 30 mile/h. Her BP is 165/95 mm Hg, HR is 105 beats per minute, and RR is 20 breaths per minute. Upon arrival, she does not open her eyes, is verbal but not making any sense, and withdraws to painful stimuli. You assign her a GCS score of 8. As you prepare to intubate the patient, a colleague notices that her left pupil has become dilated compared to the right. Which of the following has the quickest effect to reduce ICP? . Cranial decompression . Dexamethasone . Furosemide . Hyperventilation . Mannitol Paramedics bring a 5 165) A car pulls up to your ED and drops off a 19-year-old man who was shot in the chest. The man tells you his name and complains of right-sided chest pain and difficulty breathing. On primary survey, his airway is patent and his oropharynx has no blood or displaced teeth. He is breathing at 32 beats per minute with retractions and an oxygen saturation of 88% on 15 L of oxygen. There is a bullet wound to his right mid-chest with another wound in his back. His trachea is deviated to the left. On auscultation, he has diminished breath sounds on the right side. Which of the following is the most appropriate next step in management? . Stat portable chest x-ray . Intubation . Perform ED thoracotomy . Call the surgical service . Needle decompression A car pulls up to yo 166) A 43-year-old man, who currently uses drugs intravenously (IV), presents to the emergency department (ED) with 2 weeks of fever, back pain, and progressive weakness in his arms bilaterally. He reports having a cough with whitish sputum. He denies any history of recent trauma. His blood pressure (BP) is 130/75 mm Hg, heart rate (HR) is 106 beats per minute, temperature is 103°F, and respiratory rate (RR) is 16 breaths per minute. On physical examination, he has tenderness to palpation in the mid-thoracic spine, and decreased strength in the upper extremities bilaterally, with normal range of motion. Laboratory results reveal a white blood cell (WBC) count of 15,500/μL, hematocrit 40%, and platelets 225/μL. Which of the following is the most likely diagnosis? . Lung abscess . Ankylosing spondylitis . Spinal epidural abscess . Vertebral compression fracture . Spinal metastatic lesion A 43-year-old man, w 167) An 81-year-old woman is brought to the ED by her children who state that the patient is acting more tired than usual, has had fever for the last 2 days, and is more confused. Ordinarily, the patient is high functioning: she is ambulatory, cooks for herself, and walks on a treadmill 30 minutes a day. Her vital signs are BP 85/60 mm Hg, HR 125, RR 20, temperature 101.3°F, and pulse oxygenation 97% on room air. On examination, the patient has dry mucous membranes, but is otherwise unremarkable. She is oriented to person and place, but states that the year is 1925. Her laboratory results show a WBC 14,300/μL, hematocrit 31%, and platelets 350/μL. Her electrolytes are within normal limits. Blood glucose is 92 mg/dL. A chest radiograph does not show any infiltrates. Urinalysis reveals 2+ protein, trace ketones, WBC > 100/hpf, RBC 5 to 10/hpf, nitrite positive, and leukocyte esterase positive. After administering a 500 cc normal saline fluid bolus and broad- spectrum antibiotics through her peripheral IV line, the patient’s BP is 82/60 mm Hg. You suspect that the patient is in septic shock. Which of the following is the next most appropriate course of action to manage this patient with early-goal-directed therapy? . Start vasopressor therapy; repeat BP, if below a systolic of 90 mm Hg, increase the dose . Check the hematocrit and if it is less than 30% prepare to transfuse packed red blood cells (RBC) . Place a central venous line into the right internal jugular vein, measure a central venous pressure (CVP), administer normal saline boluses if the CVP is less than 8 mm Hg . Place a central venous line into the left subclavian vein, measure a CVP, administer normal saline boluses if the CVP is less than 12 mm Hg . Place a central venous line into the right femoral vein, measure a CVP, administer normal saline boluses if the CVP is less than 8 mm Hg An 81-year-old woman 168) A 23-year-old man presents to the ED with left lower-abdominal pain and left testicular pain that started 1 to 2 weeks ago and has gradually worsened. He denies nausea and vomiting. His HR is 98 beats per minute, BP is 125/65 mm Hg, temperature is 100.9°F, and RR is 18 breaths per minute. Physical examination reveals a tender left testicle with a firm nodularity on the posterolateral aspect of the testicle. Pain is relieved slightly with elevation of the testicle and the cremasteric reflex in normal. Which of the following is the next best step? . Prescribe pain medications and penicillin for coverage of syphilis. . Recommend bed rest and scrotal elevation with urology follow-up. . Attempt to untwist the left testicle by rotating it in a clockwise direction and order an immediate scrotal ultrasound. . Give ceftriaxone 250 mg intramuscularly (IM), plus a 10-day course of oral doxycycline. . Confirm the diagnosis with transillumination of the testicle, and then consult urology for surgical drainage. A 23-year-old man pr 169) A 40-year-old diabetic man presents to the ED with severe perineal pain and fever as high as 103°F. Physical examination demonstrates crepitus over the medial thigh and widespread discoloration with sharp demarcation over the scrotum. The scrotum is warm and markedly edematous. His pain appears out of proportion to the physical examination. Which of the following is the most likely diagnosis? . Epididymitis . Fournier gangrene . Scrotal edema . Paraphimosis . Testicular torsion A 40-year-old diabet 170) A 55-year-old man with a history of alcoholism and osteoarthritis developed left knee pain several days after a fall from standing height. The patient was brought to the ED by ambulance after being found on a park bench stating he was unable to walk because of the pain. On physical examination, his left knee is warm, diffusely tender, and swollen with a large effusion. He has pain on passive range of motion. His BP is 150/85 mm Hg, HR is 105 beats per minute, temperature is 102.7°F, RR is 16 breaths per minute, and fingerstick glucose is 89 mg/dL. Which of the following is the most appropriate diagnostic test? . Knee radiographs . Magnetic resonance imaging (MRI) . Erythrocyte sedimentation rate (ESR) and C-reactive protein . Arthrocentesis . Bone scan A 55-year-old man wi 171) A 35-year-old woman with systemic lupus erythematosus (SLE) is brought to the ED by her brother after he found her febrile and confused. Physical examination reveals fever, tachycardia, a waxing and waning mental status, petechiae over her oral mucosa, pallor, and mildly heme-positive stool. Her urinalysis is positive for blood, red cell casts, and proteinuria. Laboratory results reveal a blood urea nitrogen (BUN) of 40 mg/dL, and a creatinine of 2 mg/dL. Her bilirubin is elevated (unconjugated > conjugated) and her international normalized ratio (INR) is 0.98. Her complete blood count reveals WBC 12,000/μL, hematocrit 29%, and platelet count 17,000/μL with schistocytes on the peripheral smear. Which of the following is the most appropriate next step in management? . Admit to the intensive care unit (ICU) for plasmapheresis and close monitoring for acute bleeds. . Admit to the ICU for platelet transfusion and monitoring for acute bleeds. . Begin corticosteroids, transfuse platelets, and call surgery for immediate splenectomy. . Admit to the ICU for dialysis and close monitoring for acute bleeds. . Perform a noncontrast head computed tomography (CT), followed by a lumbar puncture (LP) for analysis of cerebrospinal fluid. A 35-year-old woman 172) A 30-year-old woman presents to the ED with fever, headache, a “sunburn-like” rash, and confusion. A friend states that the patient has complained of nausea, vomiting, diarrhea, and a sore throat over the past few days. Her last menstrual period began 4 days ago. Vital signs are HR 110 beats per minute, BP 80/45 mm Hg, RR of 18 breaths per minute, and temperature of 103°F. On physical examination, you note an ill-appearing female with a diffuse blanching erythroderma. Her neck is supple without signs of meningeal irritation. You note a fine desquamation of her skin, especially over the hands and feet, and hyperemia of her oropharyngeal, onjunctival, and vaginal mucus membranes. Laboratory results reveal a creatine phosphokinase (CPK) of 5000, WBC 15,000/μL, platelets of 90,000/μL, BUN 40 mg/dL, creatinine 2 mg/dL, and elevated liver enzymes. Which of the following is the most likely causative organism? . Staphylococcus aureus . Rickettsia rickettsii . Streptococcus pyogenes . Neisseria meningitidis . Neisseria gonorrhoeae A 30-year-old woman 173) A 32-year-old diabetic man steps on a nail through his running shoe. Two weeks later, he presents to the ED with fever and right foot pain. On physical examination, his heel is mildly erythematous and diffusely tender to palpation, with overlying warmth and edema. There is a small amount of purulent drainage through the puncture hole in his heel. A plain radiograph of his foot demonstrates a slight lucency of the calcaneus. He has decreased range of motion, but you are able to passively dorsiflex and plantarflex his ankle without difficulty. His vital signs include a temperature of 101.4°F, HR of 98 beats per minute, BP of 130/75 mm Hg, and RR of 16 breaths per minute. Which of the following is the most common causative organism of this condition? . Salmonella sp. . Pseudomonas aeruginosa . Staphylococcus aureus . Group.B streptococci . Pasteurella multocida A 32-year-old diabet 174) A 75-year-old woman is transferred to your ED from the local nursing home for fever, cough, and increasing lethargy. Over the past 3 days, the nursing home staff noticed increasing yellow sputum and decreasing urine output from the patient. Her BP is 118/75 mm Hg, RR is 20 breaths per minute, HR is 105 beats per minute, temperature is 100.9°F, and pulse oxygenation is 94% on room air. On examination, the patient appears dehydrated. Auscultation of the lungs reveals bibasilar crackles. Laboratory results reveal WBC 14,500/μL, hematocrit 39%, platelets 250/μL, sodium 132 mEq/L, potassium 3.5 mEq/L, chloride 100 mEq/L, bicarbonate 18 mEq/L, BUN 27 mg/dL, creatinine 1.5 mg/dL, and glucose 85 mg/dL. Serum lactate is 4.7 mmol/dL. Based on this patient’s presentation, how would you classify her condition? . Systemic inflammatory response syndrome (SIRS) . Severe systemic inflammatory response syndrome . Sepsis . Severe sepsis . Septic shock A 75-year-old woman 175) A 55-year-old man presents to the ED with fever, drooling, trismus, and a swollen neck. He reports a foul taste in his mouth since a tooth extraction 2 days ago. On physical examination, the patient appears anxious. He has bilateral submandibular swelling and elevation and protrusion of the tongue. He appears “bull-necked” with tense and markedly tender edema and brawny induration of the upper neck, and he is tender over the lower second and third molars. There is no cervical lymphadenopathy. His vital signs are: HR 105 beats per minute, BP 140/85 mm Hg, RR 26 breaths per minute, and temperature 102°F. Which of the following is the most appropriate next step in management? . Administer a dose of IV antibiotics and obtain a soft-tissue radiograph of the neck. . Administer a dose of IV antibiotics then perform an incision and drainage at the bedside. . Begin steroids to decrease inflammation and obtain an ear, nose, and throat (ENT) consult. . Discharge the patient with oral antibiotics and ENT follow-up. . Secure his airway, start IV antibiotics, and obtain an ENT consult. A 55-year-old man pr 176) A 67-year-old woman presents to the ED with a painful facial rash that has been worsening over the past 2 days. On physical examination, she has a deeply erythematous, shiny area of warm and tender skin on her left face with a sharply-demarcated and indurated border. There is minimal edema. Vitals are HR 88 beats per minute, BP 125/70 mm Hg, RR 16 breaths per minute, and temperature 101°F. Which of the following is the most appropriate next step in management? . IV antibiotics and hospital admission . Oral cephalosporin and outpatient follow-up . IV acyclovir and Tzanck smear . Systemic steroids and laboratory testing for rheumatoid factor . Dermatology consult and biopsy of the rash A 67-year-old woman 177) A 47-year-old man presents to urgent care complaining of a 2-day history of sore throat and subjective fever at home. He denies cough or vomiting. His BP is 130/75 mm Hg, HR is 85 beats per minute, temperature is 101°F, and his RR is 14 breaths per minute. He has tonsillar swelling without exudates and bilaterally enlarged and tender lymph nodes of the neck. The rapid streptococcal antigen test is negative. Which of the following is the next best step in management? . Administer penicillin and discharge the patient. . Schedule a lymph node biopsy to rule out lymphoma. . Observe for 6 hours. . Perform a throat culture, symptomatic care, and treat if results are positive. . Administer amantadine to patient and all contacts. A 47-year-old man pr 178) A 28-year-old man who just finished a 7-day course of antibiotics for pharyngitis presents to the ED with progressive difficulty swallowing. His BP is 130/65 mm Hg, HR is 95 beats per minute, temperature is 100.1°F, RR is 16 breaths per minute, and oxygen saturation is 99%. On examination, the patient is in no acute distress but has a fluctuant mass on the right side of the soft palate with deviation of the uvula. Which of the following is the most appropriate next step in management? . Needle aspiration, antibiotics, and follow-up in 24 hours . Pain control, observation for 6 hours . Admission for incision and drainage in the OR . Antibiotics and follow-up in 24 hours . CT scan, antibiotics, and follow-up in 24 hours A 28-year-old man wh 179) A 50-year-old man presents to the ED complaining of fever, sore throat, and neck pain for 24 hours. He states that 1 week ago he had two molars extracted from his mouth. His BP is 145/75 mm Hg, HR is 102 beats per minute, temperature is 101.2°F, and his RR is 16 breaths per minute. On examination you notice that the patient is drooling. There is erythema and swelling of his submandibular area that gives the appearance of a “bull neck.” His tongue is swollen and elevated and the floor of his mouth is tender. There is no fluctuant mass in his mouth. Which of the following is the most likely diagnosis? . Acute mastoiditis . Peritonsillar abscess . Ludwig angina (LA) . Acute necrotizing ulcerative gingivitis (ANUG) . Streptococcus pharyngitis A 50-year-old man pr 180) A 44-year-old man presents to the ED complaining of right foot pain. He states while playing basketball 2 weeks ago, he stepped on a nail that punctured through his sneaker and cut his great toe. He went immediately to the ED, where an x-ray was taken and confirmed negative, in addition to being administered tetanus prophylaxis. On physical examination, his toe is swollen, erythematous, and tender to palpation. There is no obvious break in the skin or abscess present. His BP is 120/75 mm Hg, HR is 80 beats per minute, temperature is 100.4°F, and RR is 16 breaths per minute. Which of the following organisms is the most likely pathogen? . Neisseria gonorrhoeae . Staphylococcus epidermidis . Sporothrix schenckii . Pseudomonas aeruginosa . Salmonella sp. A 44-year-old man pr 181) A 42-year-old diabetic man presents to the ED with 3 days of rapidly worsening scrotal and perineal pain. His HR is 115 beats per minute, BP is 135/90 mm Hg, RR is 18 breaths per minute, examination demonstrates a necrotizing infection of the scrotum and perineal subcutaneous fascia. Which of the following is the most appropriate next step in management? . 14-Day course of levofloxacin and early urology follow-up. . Oral metronidazole and cephalexin with early urology follow-up. . Pain medications, daily warm soaks, and follow-up with an urologist. . Aggressive fluid resuscitation, bedside incision and drainage, and urology consult. . Surgical debridement, broad-spectrum IV antibiotics, and hospital admission. A 42-year-old diabet 182) A 45-year-old woman presents to the ED complaining of 3 days of fever and worsening throat pain and painful odynophagia without cough or coryza. She sits on a chair, leaning forward with her mouth slightly open. She has a cup of saliva and a box of facial tissues at her side. Vitals are HR of 120 beats per minute, BP of 110/70 mm Hg, RR of 22 breaths per minute, oxygen saturation of 99% on room air, and temperature of 102°F. Her voice is hoarse, but she is able to open her mouth fully. Her posterior oropharynx is moderately hyperemic, without exudates or tonsillar enlargement. A soft tissue lateral cervical radiograph shows marked edema of the prevertebral soft tissues and absence of the vallecular space. Which of the following is the most likely diagnosis? . Retropharyngeal abscess . Peritonsillar abscess . Epiglottitis . Pharyngitis . Laryngotracheitis A 45-year-old woman 183) A 19-year-old woman presents with bilateral lower-abdominal pain, fever, nausea, vomiting, and general malaise. Her last menstrual period was 5 days ago. Vitals are HR 98 beats per minute, BP 110/65 mm Hg, RR 18breaths per minute, and temperature of 102.7°F. Pelvic examination demonstrates exquisite cervical motion tenderness and right adnexal tenderness. Laboratory reports are notable for a WBC 15,000/μL, an ESR of 95 mm/h, and a negative urine β-human chorionic gonadotropin (β-hCG). Transvaginal ultrasound demonstrates a right complex mass with cystic and solid components. Which of the following is the most appropriate next step in management? . Ceftriaxone IM plus a 14-day course of oral doxycycline and follow-up in the gynecology clinic. . Oral ofloxacin plus metronidazole and follow-up in the gynecology clinic. . Analgesics for a ruptured ovarian cyst and follow-up in the gynecology clinic. . Admission for IV antibiotics and possible laparoscopic drainage. . Admission for emergent medical or surgical treatment of an ectopic pregnancy. A 19-year-old woman 184) A 42-year-old IV drug user presents to the ED with fever, chills, pleuritic chest pain, myalgias, and general malaise. The patient’s vitals include a HR of 110 beats per minute, BP of 110/65 mm Hg, RR of 18 breaths per minute, and temperature of 103°F. Physical examination is notable for retinal hemorrhages, petechiae on the conjunctivae and mucus membranes, a faint systolic ejection murmur, and splenomegaly. Which of the following is the most likely diagnosis? . Sick sinus syndrome . Myocarditis . Pericarditis . Cardiac tamponade . Endocarditis A 42-year-old IV dru 185) A 51-year-old diabetic man complains of intense right-ear pain and discharge. On physical examination, his BP is 145/65 mm Hg, HR 91 beats per minute, and temperature 101°F. He withdraws when you retract the pinna of his ear. The external auditory canal is erythematous, edematous, and contains what looks like friable granulation tissue in the external auditory canal. The tympanic membrane is partially obstructed but appears to be erythematous, as well. Which of the following statements regarding this patient’s condition is true? . It is a common complication of otitis externa that afflicts otherwise healthy patients. . The mainstay of treatment is outpatient with oral antibiotics. . Patients are usually afebrile and have minimal ear pain. . It is caused by P aeruginosa. . Hearing loss is the most common complication. A 51-year-old diabet 186) A 26-year-old woman presents to the ED with fever, malaise, and an evolving rash in the right axilla that she initially thought was from an insect bite that she received while hiking 1 week earlier. She complains of generalized fatigue, nausea, headache, and joint pain over the past several days. Her vitals are BP of 120/75 mm Hg, HR of 75 beats per minute, RR of 16 breaths per minute, and temperature of 101°F. On physical examination, she is awake and alert, with a non-focal neurological examination. Her neck is supple, but she is diffusely tender over the shoulder, knee, and hip joints bilaterally without any distinct effusions. Her abdomen is soft and non-tender. She has a 9- cm erythematous annular plaque with a central clearing under her right axilla. Which of the following is the next best step? . Consult dermatology for a biopsy of the rash. . Perform a LP and begin treatment with IV ceftriaxone. . Prescribe doxycycline 21 days and arrange follow-up with her primary-care doctor. . Prescribe hydrocortisone cream for the rash and acetaminophen for the headache and joint pain. . Perform serologic testing for Borrelia burgdorferi and begin treatment only if positive. A 26-year-old woman 187) A 22-year-old man presents to the ED with a 3-day history of rash, fever, malaise, and mouth sores. He has been unable to eat because of mouth pain. He denies arthralgias, penile discharge, new medications, drug allergies, or prior similar episodes. On examination, the patient’s BP is 100/60 mm Hg, HR is 110 beats per minute, RR is 20 breaths per minute, and temperature is 102°F. The patient appears alert but uncomfortable. He has multiple vesiculobullous lesions on his conjunctivae and mouth as seen in the image below. Visual acuity is 20/20. Target lesions are found on his palms and soles. What is the most appropriate next step in management? . Discharge him with analgesics, antihistamines, and mouth rinses. . Discharge him with acyclovir, analgesics, antihistamines, and mouth rinses. . Discharge him after 1-L normal saline IV; and prescriptions for analgesics, antihistamines, oral prednisone, and mouth rinses. . Admit him and administer 1- to 2-L normal saline IV, oral prednisone, analgesics, antihistamines, and mouth rinses. . Admit him and administer 1- to 2-L normal saline IV; analgesics, and acyclovir IV. A 22-year-old man pr 188) A 54-year-old man with a history of hepatitis C, alcohol abuse, and cirrhotic ascites presents with increasing abdominal girth and abdominal pain. He complains of increasing difficulty breathing, especially when lying down, caused by worsening ascites. On physical examination, the patient is cachectic and appears older than his stated age. He has a diffusely tender abdomen and tense ascites. The liver is palpable 4 cm below the costal margin. Vitals include a BP of 110/65 mm Hg, HR of 110 beats per minute, RR of 22 breaths per minute, and temperature of 102°F. Which of the following is the most common organism seen in spontaneous bacterial peritonitis? . Pseudomonas aeruginosa . Enterococcus . Streptococcus pneumoniae . Enterobacteriaceae . Streptococcus viridans A 54-year-old man wi 189) A 79-year-old woman with a history of coronary artery disease who underwent a coronary artery bypass graft (CABG) surgery in 2000 is brought to the emergency department (ED) by her family for 2 days of worsening shortness of breath. For the past 2 days, she has not gotten out of bed and is confused. She does not have chest pain, fevers, or cough. Her temperature is 98.1°F, blood pressure (BP) is 85/50 mm Hg, heart rate (HR) is 125 beats per minute, and respiratory rate (RR) is 26 breaths per minute. On examination, she is unable to follow commands and is oriented only to name. The cardiovascular examination reveals tachycardia with no murmurs. Her lungs have rales bilaterally at the bases. The abdomen is soft, non-tender, and non-distended. Lower extremities have 2+ edema to the knee bilaterally. Which of the following is the most likely diagnosis? . Hypovolemic shock . Neurogenic shock . Cardiogenic shock . Anaphylactic shock . Septic shock A 79-year-old woman 190) A 32-year-old man with no past medical problems presents to the ED with palpitations. For the past 2 days he has been feeling weak and over the last 6 hours he has noticed that his heart is racing. He has no chest pain or shortness of breath. He has never felt this way before. His temperature is 98.9°F, BP is 140/82 mm Hg, HR is 180 beats per minute, and RR is 14 breaths per minute. His physical examination is normal. You obtain the following rhythm strip. What is your first-line treatment for this patient? . Synchronized cardioversion at 100 J . Adenosine 6-mg intravenous (IV) push . Adenosine 12-mg IV push . Valsalva maneuver . Verapamil 3-mg IV push A 32-year-old man wi 191) You are a passenger aboard an airplane and a 78-year-old woman is complaining of chest pain and difficulty breathing. You are the only medical professional available and volunteer to help. Fortunately, the aircraft is well-equipped with basic medical equipment, as well as with advanced cardiac life support (ACLS) medications and a cardiac monitor. On examination, the passenger’s BP is 75/40 mm Hg, HR is 180 beats per minute, and RR is 24 breaths per minute. On examination, the patient is in obvious distress, but able to answer basic questions. Her heart is tachycardic, regular, and without murmurs, rubs, or gallops. Physical examination is remarkable for a bounding carotid pulse. You attach the cardiac monitor and see a regular rhythm at 180 beats per minute with wide QRS complexes and no obvious P waves. After asking the pilot to make an emergency landing, what do you do next? . Amiodarone IV . Synchronized cardioversion . Verapamil IV . Lidocaine IV . Procainamide IV You are a passenger 192) A 41-year-old man is brought into the ED by paramedics in cardiopulmonary arrest. A friend states that the patient is a long-time user of IV heroin. You look at the monitor and see that the patient has pulseless electrical activity (PEA). Cardiopulmonary resuscitation is being performed and the patient is intubated. You decide to administer epinephrine to the patient, but realize that he does not have IV access. Which of the following drugs is ineffective when administered through an endotracheal (ET) tube? . Atropine . Naloxone . Lidocaine . Epinephrine . Sodium bicarbonate A 41-year-old man is 193) A 75-year old man complaining of chest pain is brought in to the ED by paramedics. He is barely able to speak to you because he is short of breath. The nurse immediately attaches him to the monitor, starts an IV, and gives him oxygen. His temperature is 98.9°F, BP is 70/40 mm Hg, HR is 140 beats per minute, RR is 28 breaths per minute, and oxygen saturation is 95% on room air. On examination, he is in mild distress. His heart is irregular and tachycardic. His lungs are clear to auscultation, with rales at the bases, bilaterally. An electrocardiogram (ECG) is shown below. What is your first-line treatment for this patient? . Heparin drip . Diltiazem 10 mg IV push . Metoprolol 5 mg IV push . Digoxin 0.5 mg IV . Synchronized cardioversion at 100 J A 75-year old man co 194) A 19-year-old man was struck by a motor vehicle while crossing the street. In the ED, he is awake, alert, and oriented, but complaining of severe right-leg pain. His temperature is 98.9°F, BP is 85/50 mm Hg, HR is125 beats per minute, and RR is 24 breaths per minute. You confirm that his airway is patent, breath sounds are equal bilaterally, and his abdomen is soft and nontender. His right leg is shorter than his left leg, slightly angulated, and swollen in his anterior thigh area. There is no open wound. Which of the following is the most likely diagnosis? . Hypovolemic shock . Neurogenic shock . Cardiogenic shock . Anaphylactic shock . Septic shock A 19-year-old man wa 195) You are called to the bedside of a hypotensive patient with altered mental status. The nurse hands you an ECG which shows atrial flutter at 150 beats per minute with 2:1 arteriovenous (AV) block. You feel that the patient is unstable and elect to perform emergency cardioversion. You attach the monitor leads to the patient. What is the critical next step in electrical cardioversion? . Set the appropriate energy level . Position conductor pads or paddles on patient . Charge the defibrillator . Turn on the synchronization mode . Administer 25 μg of fentanyl IV You are called to th 196) Paramedics bring in a 54-year-old man who was found down in his apartment by his wife. He is successfully intubated in the field and paramedics are currently performing cardiopulmonary resuscitation (CPR). He is transferred to an ED gurney and quickly attached to the cardiac monitors. You ask the paramedics to hold CPR and assess the patient and the rhythm strip. The monitor shows sinus bradycardia, but no pulses are palpable. On examination you appreciate bilateral breath sounds with mechanical ventilation, a soft abdomen, no rashes, and a left arm AV graft. In addition to CPR with epinephrine and atropine every 3 to 5 minutes, which intervention should be performed next? . Administer 1 ampule of sodium bicarbonate . Administer 1 ampule of calcium gluconate . Administer 1 ampule of D50 (dextrose) . Place left-sided chest tube . Perform pericardiocentesis Paramedics bring in 197) An 85-year-old man is rambling incoherently and not eating at his nursing home. Records indicate that he has a past medical history of hypertension, diabetes, dementia, and benign prostatic hypertrophy. On arrival to the ED, the patient is combative and oriented only to name. His temperature is 101°F rectally, BP is 85/50 mm Hg, HR is 125 beats per minute, RR is 22 breaths per minute, and blood sugar is 154 mg/dL. He appears uncomfortable and cachectic. His lungs are clear to auscultation, with scant crackles at the bases, and his abdomen is soft, non-tender, and non- distended. He has a Foley catheter in place draining cloudy, white urine. He has no peripheral edema. Which of the following is the most likely diagnosis? . Hypovolemic shock . Neurogenic shock . Cardiogenic shock . Anaphylactic shock . Septic shock An 85-year-old man i 198) A 72-year-old man is in the ED for the evaluation of generalized weakness over the previous 24 hours. He has a past medical history of coronary artery disease with a CABG performed 5 years ago, diabetes mellitus, and arthritis. The nurse places the patient on a cardiac monitor and begins to get his vital signs. While the nurse is obtaining the vital signs, he notices the patient suddenly becomes unresponsive. You arrive at the bedside, look at the monitor, and see the following rhythm. Which of the following is the most appropriate next step in management? . Wait 5 minutes to see if he awakens on his own. . Immediately defibrillate at 200 J (biphasic). . Perform synchronized cardioversion at 100 J. . Immediately intubate the patient. . Insert an IV line and administer amiodarone. A 72-year-old man is 199) As you arrive for your ED shift, you are called to help with a “coding” patient. The senior resident has just intubated the patient and the nurses have established IV access and attached the cardiac monitor. An emergency medical treatment (EMT) student is performing chest compressions. You ask the EMT student to stop compressions. The monitor shows a flat line with no electrical activity. You are unable to detect any pulses. What is your next step in management? . Defibrillate at 360 J . Epinephrine 1-mg IV push . Atropine 1-mg IV push . Ask the nurse to run a rhythm strip in an additional lead . Apply transcutaneous pacers As you arrive for yo 200) A 34-year-old woman with no known medical problems is having a sushi dinner with her husband. Halfway through dinner, she begins scratching her arms and her husband notices that her face is flushed. The itching intensifies and she begins to feel chest pain, shortness of breath, and dizziness. On arrival to the ED, she can barely talk. Her temperature is 100°F, BP is 85/50 mm Hg, HR is 125 beats per minute, and RR is 26 breaths per minute, and oxygen saturation is 91% on room air. Which of the following is the most likely diagnosis? . Hypovolemic shock . Neurogenic shock . Cardiogenic shock . Anaphylactic shock . Septic shock A 34-year-old woman 201) An 82-year-old nursing home patient presents to the ED in septic shock. Her BP is 75/40 mm Hg, HR is 117 beats per minute, temperature is 96.5°F, RR is 29 breaths per minute, and oxygen saturation is 87% on room air. As you perform laryngoscopy to intubate the patient, you easily visualize the vocal cords and subsequently pass the orotracheal tube through the vocal cords. You place the colorimetric end-tidal carbon dioxide device over the tube and get appropriate color change. There are equal, bilateral breath sounds on auscultation and you observe chest wall motion with ventilation. Which of the following is the most reliable method for verifying proper ET tube placement? . Chest radiograph . Visualization of the ET tube passing through the vocal cords . Observation of chest wall motion with ventilation . Hearing equal, bilateral breath sounds on auscultation . End-tidal carbon dioxide color change An 82-year-old nursi 202) An 82-year-old man presents to the ED feeling weak and dizzy. He has a past medical history of hypertension and diabetes and both are well controlled on hydrochlorothiazide, benazepril, atenolol, and metformin. On review of systems, he denies chest pain, gastrointestinal (GI) bleeding, and syncope, but states that he feels short of breath. His temperature is 98.6°F orally, BP is 86/60 mm Hg, HR is 44 beats per minute, RR is 18 breaths per minute, oxygen saturation is 98% on room air, and glucose is 116 mg/dL. He is immediately connected to the cardiac monitor. Which of the following choices best describes the ECG seen below? . Normal sinus rhythm . First-degree AV block . Second-degree Mobitz I (Wenckebach) AV block . Second-degree Mobitz II AV block . Third-degree AV block An 82-year-old man p 203) A 25-year-old man fell off his surfboard and landed on rocks. He was pulled from the water by lifeguards and brought to the ED in full cervical and spinal immobilization. He is alert and oriented to person, place, and time. He is complaining of weakness in all of his extremities. His temperature is 98.4°F, BP is 85/50 mm Hg, HR is 60 beats per minute, RR is 20 breaths per minute, and oxygen saturation is 98% on room air. On examination, he has no external signs of head injury. His heart is bradycardic without murmurs. The lungs are clear to auscultation and the abdomen is soft and non- tender. He has grossly normal peripheral sensation, but no motor strength in all four extremities. Which of the following is the most likely diagnosis? . Hypovolemic shock . Neurogenic shock . Cardiogenic shock . Anaphylactic shock . Septic shock A 25-year-old man fe 204) A 48-year-old man is brought to the ED by paramedics for generalized weakness. His medical history is significant for a CABG last month. He has been unable to get out of bed for the past day because of dizziness when changing position. He denies chest pain, shortness of breath, or syncope. His temperature is 98.9°F, BP is 86/60 mm Hg, HR is 44 beats per minute, RR is 18 breaths per minute, and oxygen saturation is 98% on room air. There is a well-healing midline sternotomy incision. Cardiac examination reveals a III/VI systolic ejection murmur. There are minimal rales at his lung bases. He is immediately attached to the cardiac monitor.His rhythm strip is shown below. What is your initial treatment? . Observe on monitor . Transcutaneous pacing . Transvenous pacing . Atropine 0.5-mg IV . Epinephrine IV drip at 2 μg/min A 48-year-old man is 205) You are caring for a 54-year-old woman with a history of schizophrenia and coronary artery disease who presents to the ED for chest pain. Her vital signs are within normal limits and her ECG is normal sinus rhythm with nonspecific ST/T wave changes. Her first troponin is sent to the laboratory and you are planning to admit her to the hospital for a complete acute coronary syndrome (ACS) evaluation. She receives aspirin and nitroglycerin and her chest pain resolves. A few minutes later, the nurse alerts you that the patient has become unconscious. You go to the bedside and find the patient awake and alert. You review the rhythm strip below. What is your next step in management? . Observe patient . Magnesium sulfate IV . Lidocaine IV . Transvenous pacemaker . Isoproterenol IV You are caring for a 206) A 48-year-old man with a medical history of cirrhosis caused by hepatitis C has been vomiting bright red blood for 1 day. On arrival to the ED, the patient is confused and unable to provide more information. His family states that he has been vomiting large amounts of bright red blood every 4 hours and has no prior history of GI bleeding. The nurses hook him up to the monitor and start two large-bore IV lines. His BP is 75/43 mm Hg, HR is 130 beats per minute, RR is 24 breaths per minute, and oxygen saturation is 98% on room air. His abdomen is soft with no masses. His rectal examination reveals bright red blood. Which type of fluid is most critical in his resuscitation? . 7% Sodium chloride . 0.9% Sodium chloride . Type and cross-matched blood . Type-specific blood . Type O, Rh-positive blood A 48-year-old man wi 207) A 19-year-old man is brought into the ED by paramedics with a stab wound to the right lower abdomen. The medics applied a pressure dressing and started an IV line en route to the hospital. On arrival, the patient has no complaints and wants to leave. His temperature is 98.4°F, BP is 130/95 mm Hg, HR is 111 beats per minute, RR is 20 breaths per minute, and oxygen saturation is 98% on room air. He is alert and oriented to person, place, and time. His abdomen is soft and nontender, with normal bowel sounds. He has a 2-cm stab wound with visible subcutaneous fat in his right lower quadrant (RLQ). You initiate the focused abdominal sonogram for trauma (FAST) examination. Which type of fluid should you start for his initial resuscitation? . 7% Sodium chloride . 0.9% Sodium chloride . 10% Albumin . Type and cross-matched blood . Type-specific blood A 19-year-old man is 208) You are notified that emergency medical service (EMS) is bringing in a patient who collapsed 5 minutes ago in his house and was intubated at the scene by paramedics. On arrival to the ED, you confirm ET placement and continue cardiopulmonary resuscitation (CPR). You connect the patient to the cardiac monitor and see the rhythm below. Which of the following is the most appropriate next step in management? . Perform synchronized cardioversion at 100 J. . Immediately defibrillate at 200 J. . Confirm the rhythm in two leads, begin CPR, then defibrillate at 200 J. . Confirm the rhythm in two leads, begin CPR, then administer amiodarone. . Confirm the rhythm in two leads, begin CPR, then administer epinephrine and atropine. You are notified tha 209) An 18-year-old college student with no past medical history presents to the ED with a diffuse rash. She also describes having a headache, fever, and arthralgias for 3 days. On examination, her temperature is 101.2°F, BP is 120/63 mm Hg, HR is 110 beats per minute, RR is 24 breaths per minute, and oxygen saturation is 98% on room air. The patient is alert and oriented to person, place, and time. She has nuchal rigidity and photophobia. Her gums are oozing blood. Her abdomen is soft and nontender and her skin has a diffuse, petechial rash. You are concerned about meningococcemia and immediately start ceftriaxone and vancomycin. Her laboratory results reveal a white blood cell (WBC) count of 13,400/μL, hematocrit 36%, platelets 80/μL, PTT 60 seconds, international normalized ratio (INR) 1.9, and fibrinogen 250 g/L. Which of the following is the most appropriate next step in management? . IV heparin . Transfuse cryoprecipitate . Transfuse packed red blood cells (RBC) . Transfuse platelets . Transfuse fresh frozen plasma (FFP) An 18-year-old colle 210) An 82-year-old man with a history of COPD and hypertension presents with shortness of breath and fever. His medications include albuterol, ipratropium, prednisone, hydrochlorothiazide, and atenolol. His temperature is 102.1°F, BP is 70/40 mm Hg, HR is 110 beats per minute, RR is 24 breaths per minute, and oxygen saturation is 91% on room air. The patient is uncomfortable and mumbling incoherently. On chest examination, you appreciate rales on the left side of his chest. His heart is tachycardic, but regular with no murmurs, rubs, or gallops. His abdomen is soft and nontender. You believe this patient is in septic shock from pneumonia and start IV fluids, broad- spectrum antibiotics, and a dopamine drip. His BP remains at 75/50 mm Hg. Which of the following is the most appropriate next step in management? . D5 normal saline IV bolus . Phenylephrine IV drip . Fludrocortisone IV . Hydrocortisone IV . Epinephrine IV drip An 82-year-old man w 211) A 64-year-old woman with a history of depression and hypertension was found down by her husband and brought in by the paramedics. Her husband says that she has recently been depressed and expressed thoughts of suicide. She usually takes fluoxetine for depression and atenolol for hypertension. On arrival, the patient is obtunded, but responds to pain and is maintaining her airway. Her temperature is 98.1°F, BP is 70/40 mm Hg, HR is 42 beats per minute, RR is 12 breaths per minute, and oxygen saturation is 94% on room air. On examination, her pupils are 3 mm and reactive bilaterally. Lungs are clear to auscultation. Heart is bradycardic, but regular, with no murmurs, rubs, or gallops. Extremities have no edema. An ECG shows first-degree AV block at 42 beats per minute, but no ST/T wave changes. Blood sugar is 112 mg/dL. What is the most specific treatment for his patient’s ingestion? . Fluid bolus . Atropine . Glucagon . Epinephrine . Cardiac pacing A 64-year-old woman 212) A 19-year-old man suffers a single gunshot wound to the left chest and is brought in by his friends. He is complaining of chest pain. On examination, his temperature is 99°F, BP is 70/40 mm Hg, HR is 140 beats per minute, RR is 16 breaths per minute, and oxygen saturation is 96% on room air. He has distended neck veins, but his trachea is not deviated. Lungs are clear to auscultation bilaterally. Heart sounds are difficult to appreciate, but you feel a bounding, regular pulse. Abdomen is soft and nontender. Extremity examination is normal. Two large-bore IV lines are placed and the patient is given 2 L of normal saline. Chest radiograph shows a globular cardiac silhouette, but a normal mediastinum and no pneumothorax. What is the definitive management of this patient? . Intubation . Tube thoracostomy . Pericardiocentesis . Thoracotomy . Blood transfusion A 19-year-old man su 213) An 84-year-old woman with a history of metastatic breast cancer presents to the ED with new- onset dyspnea and exercise intolerance for the past week. She denies fever, chest pain, or cough. On examination, her temperature is 100.3°F, BP is 70/50 mm Hg, HR is 110 beats per minute, RR is 20 breaths per minute, and oxygen saturation is 93% on room air. As your colleague is performing a physical examination, you place the portable ultrasound on the heart and see a thin echo-free area around the heart with right atrium and right ventricular collapse. Which of the following is the most likely diagnosis? . Pulmonary embolism . Congestive heart failure . Massive myocardial infarction (MI) . Cardiac tamponade . Dehydration An 84-year-old woman 214) A 50-year-old man with a history of hypertension presents to the ED with severe left-sided chest pain for 1 hour. The pain radiates down his left arm and he feels nauseated. His temperature is 98.3°F, BP is 160/92 mm Hg, HR is 92 beats per minute, RR is 16 breaths per minute, and oxygen saturation is 98% on room air. The physical examination is normal. His ECG shows ST-segment elevations in leads II, III, and aVF. You administer aspirin, nitroglycerin, and morphine sulfate and wait for his laboratory results. The nurse calls you over 10 minutes later and tells you that the patient’s BP dropped to 60/30 mm Hg with a HR of 100 beats per minute. Which of the following most likely explains his hypotension? . Medication-related adverse reaction . Cardiogenic shock . Papillary muscle rupture . Free wall rupture . Rupture of the interventricular septum A 50-year-old man wi 215) An 87-year-old woman with a history of dementia, arthritis, and hypertension presents to the ED for abdominal pain. Her caretaker reports that she is having mid-epigastric pain and had one episode of non-bloody, non-bilious vomiting prior to arrival. The patient is oriented to name only. Temperature is 99.8°F, HR is 110 beats per minute, BP is 80/44 mm Hg, RR is 16 breaths per minute, and oxygen saturation is 96% on room air. On examination, the abdomen is soft, non-tender, with no masses, rebound or guarding. Stool is brown and guaiac negative. You place two IV lines and begin fluid resuscitation. You send her blood to the laboratory and order a radiograph of her chest that is shown below. Which of the following is the cost appropriate next step in management? . Start IV antibiotics . Order a CT scan of her abdomen . Call the surgery service . Place a central venous line . Discharge home with Maalox An 87-year-old woman 216) After being fired from his job, a 35-year-old man attempts suicide by drinking from a bottle labeled “insecticide.” Three hours later, emergency medical services (EMS) brings him in to the emergency department (ED)and you notice that he is extremely diaphoretic, drooling, and vomiting. He is awake, but confused. His vital signs include a blood pressure (BP) of 170/90 mm Hg, heart rate (HR) of 100 beats per minute, respiratory rate (RR) of 22 breaths per minute, temperature of 98.6°F, and oxygen saturation of 95% on room air. Physical examination demonstrates pinpoint pupils and crackles on lung examination. What is the treatment to reverse this patient’s poisoning? . Naloxone . N-acetylcysteine (NAC) . Atropine and pralidoxime (2-PAM) . Flumazenil . Physostigmine After being fired fr 217) A 19-year-old man is brought to the ED by EMS after he was found lying on the floor at a dance club. EMS states that the patient seemed unconscious at the dance club but as soon as they transferred him onto the gurney he became combative. Upon arrival in the ED, his BP is 120/65 mm Hg, HR is 75 beats per minute, temperature is 98.9°F, RR is 12 breaths per minute, and oxygen saturation is 98% on room air. On physical examination, his pupils are midsized, equal, and reactive to light. His skin is warm and dry. Lung, cardiac, and abdominal examination are unremarkable. As you walk away from the bedside, you hear the monitor alarm signaling zero respirations and the oxygen saturation starts to drop. You perform a sternal rub and the patient sits up in bed and starts yelling at you. As you leave him for the second time, you hear the monitor alarm again signal zero respirations. You administer naloxone, but there is no change in his condition. Which of the following is most likely the substance ingested by this patient? . γ-Hydroxybutyrate (GHB) . Diazepam . Cocaine . Phencyclidine (PCP) . Heroin A 19-year-old man is 218) A 43-year-old woman presents to the ED with a 3-week history of intermittent headache, nausea, and fatigue. She was seen at her private doctor’s office 1 week ago along with her husband and children, who also have similar symptoms. They were diagnosed with a viral syndrome and told to increase their fluid intake. She states that the symptoms began approximately when it started to get cold outside. The symptoms are worse in the morning and improve while she is at work. Her BP is 123/75 mm Hg, HR is 83 beats per minute, temperature is 98.9°F, and oxygen saturation is 98% on room air. Physical examination is unremarkable. You suspect her first diagnosis was incorrect. Which of the following is the most appropriate next step to confirm your suspicion? . Order a mono spot test . Perform a nasal pharyngeal swab to test for influenza . Consult psychiatry to evaluate for malingering . Order a carboxyhemoglobin (COHb) level . Order a lead level A 43-year-old woman 219) A 47-year-old man is brought to the ED by EMS after being found wandering in the street mumbling. His BP is 150/75 mm Hg, HR is 110 beats per minute, temperature is 100.5°F, RR is 16 breaths per minute, oxygen saturation is 99% on room air, and fingerstick glucose is 98 mg/dL. On examination, the patient is confused with mumbling speech. His pupils are dilated and face is flushed. His mucous membranes and skin are dry. Which of the following toxic syndromes is this patient exhibiting? . Sympathomimetic syndrome . Anticholinergic syndrome . Cholinergic syndrome . Opioid syndrome . Ethanol syndrome A 47-year-old man is 220) A 25-year-old man is carried into the ED by two of his friends who state that he is not breathing. The patient has a history of heroin abuse. His vital signs are BP 115/70 mm Hg, HR 99 beats per minute, temperature 98.9°F, RR 3 breaths per minute, and oxygen saturation 87% on room air. You notice fresh needle marks and miotic pupils. You begin bag-valve-mask ventilation and his oxygen saturation increases to 99%. Which of the following is the most appropriate next step in management? . Continue bag-valve-mask ventilation until he breathes on his own . Endotracheal intubation of the patient . Administration of naloxone . Administration of flumazenil . Place a nasogastric tube and administer activated charcoal A 25-year-old man is 221) A 42-year-old man who is actively seizing is brought to the ED by EMS after a massive ingestion of an unknown substance. The man is known to have a history of acquired immunodeficiency syndrome (AIDS). An intravenous (IV) line is established and anticonvulsant therapy is administered. After high doses of diazepam, phenobarbital, and phenytoin, it is determined that the seizures are refractory to standard anticonvulsant therapy. Which of the following substances did this patient most likely ingest? . Cocaine . Diphenhydramine . Tricyclic antidepressant . Camphor . Isoniazid (INH) A 42-year-old man wh 222) A 60-year-old woman with a history of diabetes is brought into the ED by EMS workers who state that the patient was found on a bus in a lethargic and diaphoretic. Her fingerstick glucose level at the scene was 35 mg/dL. EMS workers quickly administered dextrose through an IV line. The patient became alert and responsive and remained this way throughout her trip to the ED. However, in the ED you notice that the patient is again diaphoretic and is mumbling her speech. Her fingerstick glucose is now 47 mg/dL. You administer dextrose and she perks right up. Which of the following diabetes medications commonly causes hypoglycemia for which the patient is likely to require hospital admission? . Regular insulin . Metformin . Glyburide . Rosiglitazone . Acarbose A 60-year-old woman 223) A 23-year-old woman presents to the ED complaining of abdominal pain, nausea, and vomiting. She has a history of depression but is not currently taking any antidepressant medications. Upon further questioning, the patient states that she ingested a bottle of pills in her medicine cabinet approximately 3 hours ago. Her BP is 115/65 mm Hg, HR is 101 beats per minute, temperature is 100.1°F, RR is 29 breaths per minute, and oxygen saturation is 100% on room air. Physical examination is unremarkable except for mild diffuse abdominal tenderness. Laboratory results reveal a white blood cell (WBC) count of 10,300/μL, hematocrit 46%, platelets 275/μL, aspartate transaminase (AST) 70 U/L, alanine transaminase (ALT) 85 U/L, alkaline phosphatase 75 U/L, sodium 143 mEq/L, potassium 3.7 mEq/L, chloride 98 mEq/L, bicarbonate 8 mEq/L, blood urea nitrogen (BUN) 22 mg/dL, creatinine 0.9 mg/dL, and glucose 85 mg/dL. Arterial blood gas values on room air are pH 7.51, PCO2 11 mm Hg, and PO2 134 mm Hg. Which of the following substances did this patient most likely ingest? . Diphenhydramine (Benadryl) . Ibuprofen . Acetaminophen . Aspirin . Pseudoephedrine A 23-year-old woman 224) A 35-year-old agitated man presents to the ED in police custody. He denies any past medical history and takes no medication. He admits to using some drugs today. His BP is 195/90 mm Hg, HR is 121 beats per minute, temperature is 100.1°F, RR is 18 breaths per minute, and oxygen saturation is 99% on room air. On examination, he is diaphoretic, and has pupils that are 8 mm in diameter, along with 3+ patella reflexes bilaterally. Electrocardiogram (ECG) reveals sinus tachycardia with a rate of 123. Which of the following toxic syndromes is this patient exhibiting? . Anticholinergic . Cholinergic . Sympathomimetic . Opioid . Serotonin A 35-year-old agitat 225) A 31-year-old woman with a known psychiatric history presents to the ED after ingesting an unknown quantity of pills from her medication vial. In the ED, she complains of nausea, abdominal cramping, and feels unsteady on her feet. On physical examination you observe that she is tremulous, ataxic, and exhibits dystonia. Which of the following substances will not be adsorbed by activated charcoal? . Digoxin . Diphenhydramine . Amitriptyline . Lithium . Acetaminophen A 31-year-old woman 226) A 27-year-old man presents to the ED extremely agitated complaining of mild chest pain and dyspnea. He states that he was snorting cocaine all afternoon. You place him on a monitor and get his vital signs. His BP is 215/130 mm Hg, HR is 112 beats per minute, temperature is 100.1°F, RR is 17 breaths per minute, and oxygen saturation is 98% on room air. An ECG reveals sinus tachycardia at a rate of 116. Which of the following is the most appropriate medication to administer? . Haloperidol . Labetalol . Esmolol . Diltiazem . Diazepam A 27-year-old man pr 227) A 30-year-old man is brought to the ED by police officers. The patient is agitated, vomiting, and complaining of body aches. He states that he is withdrawing from his medication. His vital signs are BP 160/85 mm Hg, RR 20 breaths per minute, HR 107 beats per minute, and temperature 99.7°F. On examination he is diaphoretic, has rhinorrhea, piloerection, and hyperactive bowel sounds. Which of the following substances is this patient most likely withdrawing from? . Ethanol . Cocaine . Nicotine . Methadone . Clonidine A 30-year-old man is 228) A 25-year-old man is brought into the ED by two police officers because of suspected drug use. The patient is extremely agitated and is fighting the police officers. It takes three hospital staff members and the two police officers to keep him on the stretcher. His vital signs are: BP 150/80 mm Hg, HR 107 beats per minute, temperature 99.7°F, RR 18 breaths per minute, and oxygen saturation 99% on room air. Physical examination is unremarkable except for cool, diaphoretic skin, persistent vertical and horizontal nystagmus, and occasional myoclonic jerks. Which of the following is the most likely diagnosis? . Cocaine intoxication . Cocaine withdrawal . Amphetamine intoxication . PCP intoxication . Opiate withdrawal A 25-year-old man is 229) An undomiciled 49-year-old man presents to the ED with altered mental status. His BP is 149/75 mm Hg, HR is 93 beats per minute, temperature is 97.5°F, RR is 18 breaths per minute, and O2 saturation is 99% on room air. Physical examination reveals an unkempt man with the odor of “alcohol” on his breath. His head is atraumatic and pupils are 4 mm, equal, and reactive. The neck is supple. Cardiovascular, pulmonary, and abdominal examinations are unremarkable. There is no extremity edema and his pulses are 2+ and symmetric. Neurologically, he withdraws all four extremities to deep stimuli. ECG is sinus rhythm. Laboratory results reveal: Sodium 141 mEq/L Arterial blood pH 7.26 Potassium 3.5 mEq/L Lactate 1.7 mEq/L Chloride 101 mEq/L Ethanol level undetectable Bicarbonate 14 mEq/L Measured serum osmolarity 352 mOsm/L BUN 15 mg/dL Calculated serum osmolarity 292 mOsm/kg Creatinine 0.7 mg/dL Glucose 89 mg/dL Urinalysis: no blood, ketones, or protein Which of the following statements below best describes the laboratory findings? . Anion gap metabolic acidosis and osmol gap . Anion gap metabolic acidosis without osmol gap . Nonanion gap metabolic acidosis and osmol gap . Nonanion gap metabolic acidosis without osmol gap . Metabolic alkalosis with secondary acidosis An undomiciled 49-ye 230) A 26-year-old woman, who was found lying on the floor of her apartment next to an unlabeled empty pill bottle, is brought into the ED. Her HR is 117 beats per minute, BP is 95/65 mm Hg, RR is 14 breaths per minute, and oxygen saturation is 97% on 2-L nasal cannula. On examination, the patient appears obtunded, her pupils are 3 mm and reactive. Her oropharynx is dry and there is no gag reflex to pharyngeal stimulation. Her neck is supple. The heart is tachycardic without murmurs, the lungs are clear to auscultation, and the abdomen is soft. There is normal rectal tone and brown stool that is heme negative. Her skin is cool and moist with no signs of needle tracks. Neurologically, she is unresponsive, but withdraws all extremities to deep palpation. Finger stick blood glucose is 85 mg/dL. Her ECG reveals sinus tachycardia at 119 with a QRS complex of 120 msec and a terminal R wave in lead aVR. Which of the following is the most appropriate next step in management? . Orotracheal intubation, administer activated charcoal through orogastric tube, and IV Narcan . Orotracheal intubation, administer activated charcoal through orogastric tube, and IV sodium bicarbonate . Orotracheal intubation, administer activated charcoal through orogastric tube, and IV NAC . Orotracheal intubation, administer syrup of ipecac through orogastric tube, and IV sodium bicarbonate . Induce vomiting prior to intubation to lower the risk of aspiration then administer IV sodium bicarbonate A 26-year-old woman, 231) A 37-year-old woman is brought into the ED by her friend who states that the patient swallowed approximately 50 capsules of 325-mg acetaminophen 6 hours ago in an attempted suicide. The patient states she feels nauseated and vomits while you take her history. Her BP is 100/75 mm Hg, HR is 97 beats per minute, temperature is 98.9°F, RR is 18 breaths per minute, and oxygen saturation is 99% on room air. Examination is unremarkable except for mild epigastric tenderness. Which of the following is the correct antidote for APAP overdose? . NAC . Physostigmine . Flumazenil . Naloxone . Digibind A 37-year-old woman 232) A 31-year-old man is brought to the ED by EMS who state that the man was found lying on the floor of his garage. He is rousable in the ED, speaks with slurred speech, and vomits. His BP is 140/85 mm Hg, HR is 94 beats per minute, temperature is 98.8°F, RR is 17 breaths per minute, and oxygen saturation is 99% on room air. You place an IV line, draw blood, and start a liter of normal saline running through the line. Laboratory results reveal serum sodium 139 mEq/L, potassium 3.5 mEq/L, chloride 101 mEq/L, bicarbonate 14 mEq/L, BUN 15 mg/dL, creatinine 1 mg/dL, glucose 105 mg/dL, arterial blood pH 7.27, COHb 4%, and lactate 2.8 Eq/L. Urinalysis shows: 1+ protein, trace ketones, WBC 4/hpf, red blood cell (RBC) 2 to 3/hpf, and multiple envelope-shaped and needle-shaped crystals. Which of the following conditions would best explain his metabolic acidosis? . Carbon monoxide (CO) poisoning . Ethylene glycol poisoning . Diabetic ketoacidosis . Lactic acidosis . Isopropyl alcohol poisoning A 31-year-old man is 233) A 35-year-old man who is employed as a forklift operator was found sitting outside of a warehouse. He came stumbling out complaining of dizziness and headaches. Coworkers in an adjoining warehouse also complained of headache and nausea. After collapsing outside, he regained consciousness immediately but appeared confused. In the ED, his BP is 100/54 mm Hg, HR is 103 beats per minute, temperature is 100°F, and RR is 23 breaths per minute. Physical examination is unremarkable. Laboratory results reveal WBC 10,500/μL, hematocrit 45%, platelets 110/μL, sodium 137 mEq/L, potassium 4 mEq/L, chloride 103 mEq/L, bicarbonate 21 mEq/L, BUN 8 mg/dL, creatinine 0.5 mg/dL, and glucose 89 mg/dL. Arterial blood gas results are pH 7.32, PCO2 32 mm Hg, and PO2 124 mm Hg. Which of the following is the most likely diagnosis? . Methemoglobinemia . Hypoglycemic syncope . Hydrocarbon poisoning . Opioid overdose . CO poisoning A 35-year-old man wh 234) A 51-year-old man presents to the ED complaining of nausea and abdominal pain after drinking some “bitter stuff.” He is considered one of the “regulars” who is usually at triage with ethanol intoxication. His temperature is 97.9°F, BP is 130/65 mm Hg, HR is 90 beats per minute, RR is 16 breaths per minute, and oxygen saturation is 97% on room air. Physical examination is unremarkable, except for slurred speech and the smell of acetone on the patient’s breath. Laboratory results reveal serum sodium 138 mEq/L, potassium 3.5 mEq/L, chloride 105 mEq/L, bicarbonate 23 mEq/L, BUN 10 mg/dL, creatinine 1.7 mg/dL, glucose 85 mg/dL, arterial blood pH 7.37, and lactate 1.4 mEq/L. Urinalysis shows moderate ketones. Which of the following is the most likely diagnosis? . Diabetic ketoacidosis (DKA) . Ethanol intoxication . Methanol intoxication . Isopropyl alcohol intoxication . Ethylene glycol intoxication A 51-year-old man pr 235) A 55-year-old man presents to the ED 6 hours after ingesting two bottles of his baby aspirin. He complains of nausea, vomiting, dizziness, and tinnitus. His temperature is 100.3°F, BP is 140/80 mm Hg, HR is 105 beats per minute, RR is 31 breaths per minute, and oxygen saturation is 99% on room air. Arterial blood gas on room air reveals a pH of 7.52, PCO2 10 mm Hg, and PO2 129 mm Hg. The blood salicylate level returns at 45 mg/dL. Which of the following is the most appropriate next step in management? . Administer activated charcoal, begin IV hydration, and administer sodium bicarbonate. . Administer activated charcoal, begin IV hydration, and intubate the patient for respiratory failure. . Administer activated charcoal, begin IV hydration, and administer NAC. . Arrange for immediate hemodialysis. . Observe the patient overnight to allow the body to metabolize the salicylates. A 55-year-old man pr 236) A 40-year-old man with a known history of ethanol abuse states that 2 hours ago he ingested two bottles of extra-strength Tylenol. The patient has no medical complaints except for some nausea. At 4 hours post ingestion you send blood to the laboratory to measure the serum acetaminophen concentration. The level returns and falls above the treatment line when you plot it on the APAP nomogram. You administer activated charcoal and decide to start IV NAC. Which of the following is a known adverse effect of IV NAC administration? . Hepatic failure . Anaphylactoid reaction . Hypertensive crisis . Confusion . Change in urine color A 40-year-old man wi 237) A 19-year-old woman presents to the ED with abdominal pain, nausea, vomiting, diarrhea, and hematemesis after ingesting an unknown substance in a suicide attempt. Which of the following antidotes are correctly paired? . Organophosphate—Physostigmine . Iron overdose—Deferoxamine . Aspirin overdose—NAC . Benzodiazepine overdose—Narcan . Anticholinergic overdose—Fomepizole A 19-year-old woman 238) A 34-year-old woman presents to the ED after ingesting an unknown quantity of her antidepressant pills. EMS workers found an empty bottle of amitriptyline on her apartment floor. She is awake but appears delirious. Her BP is 130/65 mm Hg, HR is 101 beats per minute, temperature is 99.1°F, RR is 16 breaths per minute, and oxygen saturation is 100% on room air. On examination, her pupils are 7 mm and reactive. Her face is flushed and mucous membranes are dry. Her lungs are clear and heart is without murmurs. The abdomen is soft, nontender, and with decreased bowel sounds. She is moving all four extremities. ECG reveals sinus rhythm at a rate of 99 and QRS just under 100 msec. In a tricyclic antidepressant (TCA) overdose, which of the following is responsible for her mydriasis, dry mucous membranes, and delirium? . Sodium channel blockade . Muscarinic receptor blockade . Inhibition of serotonin and norepinephrine reuptake . Histamine receptor blockade . α-Receptor blockade A 34-year-old woman 239) You receive notification from EMS that they are bringing in a 17-yearold male who was found unconscious by a police officer. The police officer at the scene states that he snuck up on a group of kids that he thought were using drugs. Two of them got away and one just fell to the ground seconds after standing up. Lying on the ground next to the adolescent were plastic bags. The emergency medical technician states that the patient was in ventricular fibrillation. He was shocked in the field and is now in a sinus rhythm. The EMT also administered IV dextrose, thiamine, and naloxone without any change in mental status. Which of the following substances was the patient most likely abusing? . Butane . Ethanol . Heroin . Cocaine . PCP You receive notifica 240) A 61-year-old man with a history of depression and hypertension is brought to the ED by EMS for altered mental status. The patient’s wife states that he stopped taking his fluoxetine 1 month ago and now only takes metoprolol for his hypertension. The patient’s BP is 75/40 mm Hg, HR is 39 beats per minute, RR is 14 breaths per minute, oxygen saturation is 99% on 100% oxygen, and fingerstick glucose is 81 mg/dL. The patient is awake and moaning, responding only to deep stimuli. You suspect an overdose of metoprolol. You endotracheally intubate the patient for airway control. Which of the following is the most appropriate next step in management? . Syrup of ipecac, normal saline bolus, epinephrine . Cardioversion with 200 J then administer epinephrine . Cardioversion with 200 J then administer atropine . Normal saline bolus, atropine, epinephrine, and activated charcoal . Normal saline bolus, atropine, glucagon, and activated charcoal A 61-year-old man wi 241) A 22-year-old woman presents to the ED by ambulance from a dance club. The paramedics report that the patient was agitated in the club and had a generalized seizure. Her BP is 165/100 mm Hg, HR is 119 beats per minute, temperature is 100.9°F, RR is 17 breaths per minute, oxygen saturation is 98% on room air, and finger stick glucose is 92 mg/dL. On examination, the patient is hyperactive and appears to be hallucinating. Her pupils are dilated to 6 mm bilaterally and reactive. Her neck is supple. Examination of the heart is unremarkable except for tachycardia. Her lungs are clear and abdomen is soft and non-tender. The patient moves all four extremities. Laboratory results are as follows: Sodium 109 mEq/L WBC 12,000/mm3 Potassium 3.5 mEq/L Hct 49% Chloride 83 mEq/L Platelets 350/μL Bicarbonate 20 mEq/L BUN 10 mg/dL Creatinine 1 mg/dL Glucose 103 mg/dL Which of the following substances did this patient most likely consume? . Cocaine . Heroin . 3,4-Methylenedioxymethamphetamine (MDMA) . Ketamine (special K) . PCP A 22-year-old woman 242) An asymptomatic young adult was brought to the ED by a police officer after his home was raided. The patient swallowed five small packets of an unknown substance before being arrested. His BP is 125/75 mm Hg, HR is 85 beats per minute, temperature is 98.7°F, and RR is 16 breaths per minute. Physical examination is unremarkable. An abdominal radiograph confirms intraluminal small bowel densities. Which of the following is the most appropriate treatment? . Magnesium citrate . Gastric lavage . Activated charcoal and polyethylene glycol . Syrup of ipecac . NAC An asymptomatic youn 243) A 33-year-old woman presents to the ED with a painful sprained ankle. She has a past medical history of depression for which she is taking phenelzine, a monoamine oxidase inhibitor. After you place an elastic wrap on her ankle, she asks you to prescribe her some pain medication. Which of the following medications is contraindicated in patients taking a monoamine oxidase inhibitor? . Ibuprofen . Acetaminophen . Meperidine . Oxycodone . Hydrocodone A 33-year-old woman 244) A 27-year-old woman presents to the ED 6 hours after the onset of body aches, abdominal cramping, and diarrhea. She is currently visiting relatives and normally lives in another state. She regularly takes six to eight tablets daily of hydrocodone for chronic low-back pain, sumatriptan for migraines, and amitriptyline and paroxetine for bulimia nervosa. Her BP is 130/80 mm Hg, HR is 100 beats per minute, temperature is 98.6°F, RR is 16 breaths per minute, and oxygen saturation is 99% on room air. Examination shows diaphoresis, dilated pupils, and piloerection. Neurologically she is moving all four extremities and you do not note tremors. She is alert and cooperative but seems restless. She denies hallucinations or suicidal ideations. She becomes very angry when you ask her for the phone numbers of her regular physicians. Which of the following is the most likely explanation of her symptoms? . Anticholinergic overdose . TCA intoxication . Ethanol withdrawal . Serotonin syndrome . Opiate withdrawal A 27-year-old woman 245) A 69-year-old woman with a past medical history of hypertension, hypercholesterolemia, diabetes mellitus type 1, and alcohol abuse is brought to the emergency department (ED) by her daughter who states that her mom has been acting funny over the last hour. She states that the patient did not know where she was despite being in her own house. She also did not recognize her family and was speaking incomprehensibly. Her blood pressure (BP) is 150/80 mm Hg, heart rate (HR) is 90 beats per minute, temperature is 98.9°F, and her respiratory rate (RR) is 16 breaths per minute. On physical examination she is diaphoretic, agitated, and tremulous. Electrocardiogram (ECG) is sinus rhythm with normal ST segments and T waves. Which of the following is the most appropriate course of action for this patient? . Administer a benzodiazepine to treat her ethanol withdrawal. . Activate the stroke team and bring the patient directly to the computed tomographic (CT) scanner. . Get a stat fingerstick and administer dextrose if her blood sugar is low. . Request a psychiatric consult for probable sundowning. . Administer haloperidol for sedation. A 69-year-old woman 246) A 74-year-old lethargic woman is brought to the ED by her family. Her daughter states that the patient has been progressively somnolent over the last week and could not be woken up today. The patient takes medications for diabetes, hypertension, hypothyroidism, and a recent ankle sprain, which is treated with a hydrocodone/acetaminophen combination. In the ED, the patient is profoundly lethargic, responsive only to pain, and has periorbital edema and delayed relaxation of the deep tendon reflexes. Her BP is 145/84 mm Hg, HR is 56 beats per minute, temperature is 94.8° F, and RR is 12 breaths per minute. Which of the following is the most likely diagnosis? . Hypoglycemia . Opioid overdose . Stroke . Myxedema coma . Depression A 74-year-old lethar 247) A 79-year-old man presents to the ED by paramedics with the chief complaint of agitation and confusion over the previous 12 hours. He has a past medical history of schizophrenia and is not taking any of his antipsychotics. His BP is 135/85 mm Hg, HR is 119 beats per minute, RR is 18 breaths per minute, oxygen saturation is 97% on room air, and fingerstick glucose is 135 mg/dL. Because of his agitation at triage, he was placed in wrist restraints. At this time, he is calm but confused. Examination reveals warm and clammy skin and 4-mm pupils that are equal and reactive. His cardiac examination reveals tachycardia and no murmurs. His lungs are clear to auscultation and his abdomen is soft and nontender. He is able to move all of his extremities. Which of the following is the most appropriate next step in management? . Administer haloperidol or lorazepam . Consult psychiatry . Order a CT scan of his head . Send a urine toxicologic screen . Obtain a rectal temperature A 79-year-old man pr 248) A 25-year-old man is brought to the ED by emergency medical service (EMS) accompanied by his girlfriend who reports that the patient had a seizure 30 minutes ago and is still confused. The girlfriend reports that the patient is a known epileptic who has been doing well on his latest medication regimen. The exact seizure medications are unknown. On arrival to the ED, the patient develops continuous clonic movements of his upper and lower extremities. The patient’s vital signs are BP of 162/85 mm Hg, HR of 110 beats per minute, and pulse oximetry of 91% on room air. Capillary glucose level is 95 mg/dL. Which of the following is the most appropriate next step in management? . Place the patient in a lateral decubitus position. . Administer lorazepam. . Administer phenytoin. . Perform rapid sequence intubation on the patient. . Look up the patient’s medical records and administer his current antiepileptic regimen. A 25-year-old man is 249) A 19-year-old college student presents to the ED complaining of headache, sore throat, myalgias, and rash that developed over the previous 12 hours. Her BP is 95/60 mm Hg, HR is 132 beats per minute, temperature is 103.9°F, RR is 19 breaths per minute, and oxygen saturation is 98% on room air. She is confused and oriented only to person. Physical examination is remarkable for pain with neck flexion, a petechial and purpuric rash on her extremities, and delayed capillary refill. Which of the following best describes the emergency physicians’ priority in managing this patient? . Collect two sets of blood cultures prior to antibiotic administration. . Call the patient’s parents and have them come immediately to the hospital. . Call her roommate to gather more information. . Begin fluid resuscitation, administer intravenous (IV) antibiotics, and perform a lumbar puncture (LP). . Administer acetaminophen to see if her headache and fever resolve. A 19-year-old colleg 250) A 21-year-old college student is brought to the ED by her roommate who states that the patient has been very sleepy today. She has a history of diabetes and has not refilled her medication in over a week. Her BP is 95/61 mm Hg, HR is 132 beats per minute, temperature is 99.7°F, and RR is 20 breaths per minute. Her fingerstick glucose is 530 mg/dL. Which of the following choices most closely matches what you would expect to find on her arterial blood gas with electrolytes and urinalysis? . pH 7.38, anion gap 5, normal urinalysis . pH 7.57, anion gap 21, presence of glucose and leukocytes in urine . pH 7.47, anion gap 12, presence of glucose and ketones in urine . pH 7.26, anion gap 12, presence of glucose and ketones in urine . pH 7.26, anion gap 21, presence of glucose and ketones in urine A 21-year-old colleg 251) A 65-year-old actively seizing woman is brought to the ED by EMS. She was found slumped over at the bus stop bench. EMS personnel state that when they found the woman she was diaphoretic and her speech was garbled. En route to the hospital, she started to seize. As you wheel her to a room, the nurse gives you some of her vital signs which are a BP of 150/90 mm Hg, HR of 115 beats per minute, and oxygen saturation of 96%. Which of the following is the next best step in managing this patient? . Request a rectal temperature to rule out meningitis. . Call the CT technologist and tell them you are bringing over a seizing patient. . Ask for a stat ECG and administer an aspirin. . Check the patient’s fingerstick blood glucose level. . Intubate the patient. A 65-year-old active 252) A 48-year-old man presents to the ED with ethanol intoxication. His BP is 150/70 mm Hg, HR is 95 beats per minute, temperature is 97.9°F, RR is 14 breaths per minute, and oxygen saturation is 93% on room air. The patient is somnolent and snoring loudly with occasional gasps for air. On examination, the patient’s gag reflex is intact, his lungs are clear to auscultation, heart is without murmurs, and abdomen is soft and nontender. He is rousable to stimulation. A head CT is negative for intracranial injury. His ethanol level is 270 mg/dL. Which of the following actions is most appropriate to assist the patient with respirations? . Nasal airway . Oral airway . Bag-valve-mask ventilation . Laryngeal mask airway . Tracheoesophageal airway A 48-year-old man pr 253) A 52-year-old woman is brought to the ED by her husband for altered mental status for 1 day. The patient has hypertension and diabetes but has not been taking her medications for the last 5 days since she lost her insurance and could not afford her prescriptions. Her BP is 168/91 mm Hg, HR is 125 beats per minute, temperature is 99.8°F, and RR is 18 breaths per minute. Her fingerstick glucose is 900 mg/dL. There is glucose in her urine, but no ketones. Which of the following is the most appropriate next step in management? . Administer IV fluids and insulin . Obtain head CT scan . Obtain ECG . Obtain chest radiograph and urine culture . Administer broad coverage antibiotics A 52-year-old woman 254) A 45-year-old man is brought to the ED by his coworkers after collapsing to the floor while at work. A coworker states that the patient mistakenly took several tablets of his oral diabetic medications a few hours ago. The patient is unresponsive and diaphoretic. His BP is 142/78 mm Hg, HR is 115 beats per minute, temperature is 98.9°F, and RR is 12 breaths per minute. A bedside glucose reads 42 mg/dL. Which of the following is the most appropriate management of this patient? . Administer IV dextrose, obtain a repeat fingerstick glucose every hour and if normal after 6 hours discharge the patient home. . Administer IV dextrose and continue monitoring his blood sugar for at least 24 hours. . Administer IV fluids and insulin. . Administer IV fluids. . Administer activated charcoal and IV fluids. A 45-year-old man is 255) A 59-year-old man is brought into the ED accompanied by his son who states that his father is acting irritable and occasionally confused. The son states that his father has a history of hepatitis from a transfusion he received many years ago. Over the past 5 years, his liver function slowly deteriorated. His vital signs include BP of 145/80 mm Hg, a HR of 78 beats per minute, RR of 16 breaths per minute, oxygen saturation of 98%, and temperature of 98°F. Laboratory results are all within normal limits, except for an ammonia level that is significantly elevated. Which of the following is the best therapy? . Vancomycin and gentamycin . Lactulose and neomycin . Ampicillin and gentamycin . Levofloxacin . Ciprofloxacin A 59-year-old man is 256) A 40-year-old man who is an employee of the hospital is brought to the ED actively seizing. A coworker states that the patient has a known seizure disorder and currently takes phenytoin for the disorder. He also tells you that the patient has been under stress recently and may not have taken his last few doses of medication. You call for the nurse to place a face mask with 100% oxygen and gain IV access. You then ask for a medication to be drawn up. Which is the most appropriate initial medication you should administer in this actively seizing patient? . Phenytoin . Diazepam . Phenobarbital . Valproic acid . Lithium A 40-year-old man wh 257) A 32-year-old gravida 1, para 1 who gave birth by normal vaginal delivery at 38-weeks gestation 2 days ago presents to the ED complaining of bilateral hand swelling and severe headache that started 2 hours ago. Her BP is 187/110 mm Hg, HR is 85 beats per minute, temperature is 97.5° F, and RR is 15 breaths per minute. Urinalysis reveals 3+ protein. As you are examining the patient, she proceeds to have a generalized tonic-clonic seizure. Which of the following is the most appropriate next step in management? . Administer magnesium sulfate IV. . Administer labetalol to reduce her BP and morphine sulfate to address her headache. . Administer sumatriptan and place the patient into a dark quiet room. . Administer a loading dose of phenytoin, order a head CT scan, and call for a neurology consult. . Administer diazepam and normal saline IV. A 32-year-old gravid 258) An unconscious 51-year-old woman is brought to the ED by EMS. A friend states that the patient was complaining of feeling weak. She vomited and subsequently “blacked out” in the ambulance. The friend states that the patient has no medical problems and takes no medications. She also states that the patient smokes cigarettes and uses cocaine, and that they were snorting cocaine together prior to her blacking out. The patient’s BP is 195/80 mm Hg, HR is 50 beats per minute, temperature is 98.6°F, and RR is 7 breaths per minute. What is the eponym associated with her vital signs? . Cushing syndrome . Cushing reflex . Cullen sign . Charcot triad . Chvostek sign An unconscious 51-ye 259) A 47-year-old man is brought to the ED by EMS after being persistently agitated at a business meeting. The patient’s coworkers state that he has been working nonstop for a day-and-a-half and that he always seemed like a healthy guy who frequented bars every night. EMS administered 25 g of dextrose and thiamine with no symptom improvement. In the ED, the patient is anxious, confused, tremulous, and diaphoretic. He denies any medical problems, medications, or drug ingestions. His BP is 182/92 mm Hg, HR is 139 beats per minute, temperature is 100.4°F, RR is 18 breaths per minute, and fingerstick glucose is 103 mg/dL. An ECG reveals sinus tachycardia. Which of the following is the next best step? . Administer acetaminophen . Administer folate . Administer diazepam . Recheck fingerstick glucose . Administer labetalol A 47-year-old man is 260) A 65-year-old man presents to the ED with a headache, drowsiness, and confusion. He has a history of long-standing hypertension. His BP is 230/120 mm Hg, pulse 87 beats per minute, RR 18 breaths per minutes, and oxygen saturation 97% on room air. On examination, you note papilledema. A head CT scan is performed and there is no evidence of ischemia or hemorrhage. Which of the following is the most appropriate method to lower his BP? . Administer propofol for rapid reduction in BP. . Administer mannitol for rapid reduction in BP and intracranial pressure (ICP). . Administer a high-dose diuretic to reduce preload. . Administer labetalol until his BP is 140/80 mm Hg. . Administer labetalol until his BP is 180/100 mm Hg. A 65-year-old man pr 261) A 74-year-old woman is brought to the ED by EMS for altered mental status. Her BP is 138/72 mm Hg, HR is 91 beats per minute, RR is 17 breaths per minute, and temperature is 100.9°F. A head CT is normal. LP results revealed the following: WBC 1020/mL with 90% polymorphonuclear cells Glucose 21 mg/dL Protein 225 g/L Which of the following is the most likely diagnosis? . TB meningitis . Bacterial meningitis . Viral meningitis . Fungal meningitis . Encephalitis A 74-year-old woman 262) A 67-year-old man presents to the ED for worsening confusion. His wife states that he received his first dose of chemotherapy for lung cancer 2 days ago. Over the last 24 hours, the patient became confused. His BP is 130/70 mm Hg, HR is 87 beats per minute, and temperature is 98.9°F. While in the ED, the patient seizes. You administer an antiepileptic and the seizure immediately stops. You compare his current electrolyte panel to one taken 2 days ago. Two days ago Today Sodium (mEq/L) 139 113 Potassium (mEq/L) 4.1 3.9 Chloride (mEq/L) 105 98 Bicarbonate (mEq/L) 23 20 BUN (mg/dL) 13 17 Creatinine (mg/dL) 0.4 0.7 Glucose (mg/dL) 98 92. Which of the following is the most appropriate treatment? . 0.45% saline . 0.9% saline . 3% saline . 5% dextrose . 50% dextrose A 67-year-old man pr 263) A 31-year-old woman with a history of schizophrenia presents to the ED for altered mental status. A friend states that the patient is on multiple medications for her schizophrenia. Her BP is 150/80 mm Hg, HR 121 beats per minute, RR 20 breaths per minute, and temperature 104.5°F. On examination, the patient is diaphoretic with distinctive “lead-pipe” rigidity of her musculature. You believe the patient has neuroleptic malignant syndrome. After basic stabilizing measures, which of the following medications is most appropriate to administer? . Haloperidol . Droperidol . Dantrolene . Diphenhydramine . Acetaminophen A 31-year-old woman 264) A 56-year-old man is brought in from the homeless shelter for strange, irrational behavior and unsteady gait for 1 day. A worker at the shelter reports that the patient is a frequent abuser of alcohol. On examination, the patient is alert but oriented to name only and is unable to give full history. He does not appear clinically intoxicated. You note horizontal nystagmus and ataxia. What is the most likely diagnosis? . Wernicke encephalopathy . Korsakoff syndrome . Normal pressure hydrocephalus . Central vertigo . Alcohol withdrawal A 56-year-old man is 265) An 18-year-old girl is brought to the ED from a party for agitation and attacking her boyfriend with a knife. Her boyfriend admits that she had several liquor shots and used intranasal cocaine at the party prior to becoming agitated, paranoid, and attacking him. Her BP is 145/80 mm Hg, HR is 126 beats per minute, temperature is 100.8°F, and RR is 20 breaths per minute. The patient is agitated, screaming, and resisting examination. What is the next best step in the management of this patient? . IV β-blocker . IV benzodiazepine . Acetaminophen . Lithium . Drug abuse specialist consult An 18-year-old girl 266) A 78-year-old woman is transferred from a nursing home with altered mental status and fever. The nursing home reports that the patient was febrile to 102.3°F, disoriented, confused, and incontinent of urine. Her past medical history includes hypertension, a stroke with residual right- sided weakness, and night time agitation for which she was started on haloperidol 3 days ago. Her BP is 215/105 mm Hg, HR is 132 beats per minute, temperature is 102.8°F, and RR is 20 breaths per minute. On examination, the patient is riented to name only, tremulous, diaphoretic and has marked muscular rigidity and three out of five right upper and lower extremity strength. What is the most likely diagnosis? . Urinary tract infection . Malignant hyperthermia . Neuroleptic malignant syndrome Page 79 of 80 . Recurrent stroke . Meningoencephalitis A 78-year-old woman 267) A 54-year-old man is brought to the ED by his wife for bizarre behavior. The wife complains that her husband has not been acting like his usual self over the last several days. She states that he has not had any change in sleep, appetite, or activity level. She also recalls that her husband complained of morning headaches for the last 2 months. The patient is otherwise in good health and does not take any medications. His BP is 135/87 mm Hg, HR is 76 beats per minute, temperature is 98.9°F, and RR is 14 breaths per minute. His examination is otherwise unremarkable. Which of the following is the most likely diagnosis? . Migraine headache . Tension headache . Subarachnoid hemorrhage . Pseudotumor cerebri . Frontal lobe mass A 54-year-old man is 268) A 27-year-old woman is brought to the ED by her husband after having a first-time seizure at home. She has no past medical history, and had no complications while delivering her newborn vaginally 1 week prior to presentation. In the ED, her BP is 178/95 mm Hg, HR is 97 beats per minute, temperature is 99.1°F, and RR is 18 breaths per minute. On examination, she has mild edema of her hands and feet. The seizure stopped spontaneously, but the patient is postictal appearing and cannot answer your questions. Which of the following is the most appropriate diagnostic test? . Complete blood count . Head CT . LP . Urinalysis . ECG A 27-year-old woman 269) A 46-year-old woman is brought to the ED by her husband for 1 day of worsening confusion. The patient has a history of systemic lupus erythematosus (SLE) and takes chronic oral steroids. She has not been feeling well for the last few days. Her BP is 167/92 mm Hg, HR is 95 beats per minute, temperature is 100.3°F, and RR is 16 breaths per minute. On examination the patient is oriented to name and has diffuse petechiae on her torso and extremities. Laboratory results reveal hematocrit 23%, platelets 17,000/μL, BUN 38 mg/dL, creatinine 1.9 mg/dL. Which of the following is the most likely diagnosis? . Henoch-Schönlein purpura . Disseminated intravascular coagulopathy . Von Willebrand disease . Idiopathic thrombocytopenic purpura . Thrombotic thrombocytopenic purpura (TTP) A 46-year-old woman 270) A 63-year-old man presents to the ED complaining of headache, vomiting, and “not being able to think straight” for 1 day. The patient states that he has hypertension and diabetes but ran out of his medications in the last week. His BP is 245/138 mm Hg, HR is 90 beats per minute, temperature is 98.7°F, and his RR is 14 breaths per minute. Fingerstick glucose is 178 mg/dL. On examination the patient appears slightly confused and oriented to name and place only. The neurologic examination is significant for papilledema. Which of the following is the most appropriate next step in management? Page 80 of 80 . Nitroprusside IV . Magnesium sulfate IV . Metoprolol by mouth . Hydrochlorothiazide by mouth . Obtain head CT A 63-year-old man pr 271) A 26-year-old man with a long history of epilepsy is brought to the ED for a recent seizure. While in the ED, he is rhythmically moving his right leg and is unresponsive. Which of the following best describes this seizure pattern? . Petit mal seizure . Generalized tonic-clonic seizure . Partial seizure with secondary generalization . Simple partial seizure . Complex partial seizure A 26-year-old man wi 272) A 58-year-old woman is brought in to the ED after a witnessed syncopal event. Upon arrival, the patient appears confused and agitated. Her vitals include HR of 89 beats per minute, BP of 145/70 mm Hg, RR of 16 breaths per minute, and oxygen saturation of 98% on room air. Within a few minutes, the patient is more alert and oriented. She denies any chest pain, headache, abdominal pain, or weakness preceding the event and is currently asymptomatic. She also states that she has not taken her antiepileptic medications in 2 days. The patient’s examination is unremarkable including a nonfocal neurological examination. Given this patient’s history and evolving examination, what is the most likely etiology of this patient’s syncopal event? . Cerebrovascular accident . Transient ischemic attack . Seizure . Aortic dissection . Pulmonary embolus A 58-year-old woman [Show More]

Last updated: 1 year ago

Preview 1 out of 81 pages

Add to cart

Instant download

We Accept:

We Accept
document-preview

Buy this document to get the full access instantly

Instant Download Access after purchase

Add to cart

Instant download

We Accept:

We Accept

Reviews( 0 )

$9.50

Add to cart

We Accept:

We Accept

Instant download

Can't find what you want? Try our AI powered Search

OR

REQUEST DOCUMENT
90
0

Document information


Connected school, study & course


About the document


Uploaded On

May 05, 2020

Number of pages

81

Written in

Seller


seller-icon
James

Member since 4 years

165 Documents Sold


Additional information

This document has been written for:

Uploaded

May 05, 2020

Downloads

 0

Views

 90

Document Keyword Tags

What is Browsegrades

In Browsegrades, a student can earn by offering help to other student. Students can help other students with materials by upploading their notes and earn money.

We are here to help

We're available through e-mail, Twitter, Facebook, and live chat.
 FAQ
 Questions? Leave a message!

Follow us on
 Twitter

Copyright © Browsegrades · High quality services·